Pathology Flashcards

1
Q

Why male patient with atherosclerosis, intermittent claudication, could have sexual dysfunction?

A

Aortoiliac atherosclerosis ▶️ ⬇️ blood flow to internal pudendal branches of internal iliac ▶️ hard sustain erection

How well did you know this?
1
Not at all
2
3
4
5
Perfectly
2
Q

Medium-sized arteries with calcification of internal elastic lamina and media, NOT obstruct blood flow, NOT involve intima. Disease and finding in x-ray.

A

Mönckeberg sclerosis

“Pipestem appearance”

How well did you know this?
1
Not at all
2
3
4
5
Perfectly
3
Q

Complication of septic abortion

A

Asherman’s syndrome ▶️ synechiae (adhesions) in uterine cavity ▶️ secondary amenorrhea and infertility

How well did you know this?
1
Not at all
2
3
4
5
Perfectly
4
Q

Principal site in kidney where uric acid precipitate. Why?

A

Distal tubules and collecting ducts

Segments with more acidic environment ▶️ uric acid crystals precipitate in acid pH.

How well did you know this?
1
Not at all
2
3
4
5
Perfectly
5
Q

How could the chemotherapy for a high grade lymphoma, leukemia or other rapid-cell turnover tumor lead in obstructive uropathy and renal failure?

A

Tumor lysis syndrome ▶️ ⬆️ uric acid ▶️ precipitate in uric acid crystals

How well did you know this?
1
Not at all
2
3
4
5
Perfectly
6
Q

Achalasia associate with which type of cancer? Secondary causes of achalasia.

A
  • Esophageal cancer ▶️ squamous cell carcinoma

- Secondary ▶️ chagas disease (T. cruzi), extraesophageal malignancies (mass effect or paraneoplastic)

How well did you know this?
1
Not at all
2
3
4
5
Perfectly
7
Q

What suggest an increase in acetylcholinesterase level in amniotic fluid in 25th week of pregnancy? What other marker you can find?

A
  • Neural tube defects ▶️ failure of fusion in week 4th

- ⬆️ alfa-fetoprotein (also elevated in maternal plasma because crosess the placenta)

How well did you know this?
1
Not at all
2
3
4
5
Perfectly
8
Q

Posterior neural tube defects. In which of them the alfa-fetoprotein is normal?

A

Spina bifida oculta (AFP normal), meningocele, myelomeningocele

How well did you know this?
1
Not at all
2
3
4
5
Perfectly
9
Q

Hirschsprung’s disease is an example of which fetal abnormality?

A

Migration abnormality ▶️ failure neural crest cells to migrate to intestinal wall

How well did you know this?
1
Not at all
2
3
4
5
Perfectly
10
Q

Duodenal atresia is an example of which fetal abnormality?

A

Abnormal apoptosis ▶️ duodenal lumen fails to recanalize at 8-10 weeks of gestation

How well did you know this?
1
Not at all
2
3
4
5
Perfectly
11
Q

How is the Retinoblastoma protein phosphorylated and its function?

A
  • Growth factors ▶️ (+) cyclin D, cyclin E, CDK (cyclin kinases) 4, 8
  • G1-S progression ▶️ ⬆️ dihydrofolate reductase, DNA polymerase ▶️ enzymes for the S phase (DNA synthesis)
How well did you know this?
1
Not at all
2
3
4
5
Perfectly
12
Q

Function of p27, levels on normal and tumoral tissue

A
  • Cell cycle inhibitor ▶️ (-) cyclin-dependent kinases ▶️ arrest cell in G1
  • Normal tissue - high levels
  • Tumoral tissue - low levels
How well did you know this?
1
Not at all
2
3
4
5
Perfectly
13
Q

When do you use an inferior vena cava filter?

A

Prevent progression of DVT from legs to lungs vasculature when anticoagulation is contraindicated

How well did you know this?
1
Not at all
2
3
4
5
Perfectly
14
Q

Test for diagnose zollinger-ellison syndrome. Association with which syndrome?

A
  • Positive secretin stimulation test: gastrin remain ⬆️ after secretin administration (normally ⬇️ gastrin release)
  • MEN 1
How well did you know this?
1
Not at all
2
3
4
5
Perfectly
15
Q

Why do zollinger-ellison patients have diarrhea?

A

Malabsorption ▶️ pancreatic/intestinal enzymes inactivated by gastric acid

How well did you know this?
1
Not at all
2
3
4
5
Perfectly
16
Q

Location and clinical manifestation of VIPoma.

A

Neuroendocrine tumor of pancreas ▶️ ⬆️ Vasoactive intestinal polypeptide ▶️ diarrhea persist with fasting, achlorhydria, hypokalemia

How well did you know this?
1
Not at all
2
3
4
5
Perfectly
17
Q

Histologic finding of Krukenberg tumor. What is this tumor?

A
  • Signet ring cells - mucin producers ▶️ large amount of mucin displacing the nucleus
  • Metastatic tumor in ovaries from diffuse gastric carcinoma (linitis plastica) or breast cancer
How well did you know this?
1
Not at all
2
3
4
5
Perfectly
18
Q

Triad of renal cell carcinoma. Histologic finding. What is the most common presentation?

A
  • Hematuria, flank pain, palpable abdominal mass (minority patients <10%)
  • Most common presentation→Non-specific symptoms (fever, weight-loss, anorexia, etc) and paraneoplastic syndromes (ex, polycythemia - ↑EPO; hypercalcemia - ↑PTH)
  • Lungs most common metastases
  • Nests of (proximal tubular) epithelial cells (rounded, polygonal) with abundant clear cytoplasm→↑glycogen and lipids
    FRAGMENT CARD!

*Most common renal malignancy→clear cell carcinoma

How well did you know this?
1
Not at all
2
3
4
5
Perfectly
19
Q

Virtual pathognomonic finding of cystic fibrosis and its consequence. Why does it happen?

A
  • Congenital bilateral agenesia of vas deferens (CBAVD)▶️ Azoospermia (unable secrete semen) ▶️ infertility (🚫 sperm transport, normal spermatogenesis)
  • CFTR mutation ▶️ abnormal development wolffian structures
How well did you know this?
1
Not at all
2
3
4
5
Perfectly
20
Q

Diagnosis of cystic fibrosis.

A
  • ⬆️ Chloride levels in sweat ▶️ Cl channel in sweat glands can not reabsorb the Cl
  • Abnormal nasal transepithelial potential difference
  • Genetic testing→defect CFRT gene on chromosome 7, deletion Phe508
How well did you know this?
1
Not at all
2
3
4
5
Perfectly
21
Q

Why can the gastroesophageal reflux disease (GERD) course with dysphagia?

A

Impaired peristalsis, esophageal inflammation, stricture or malignancy

How well did you know this?
1
Not at all
2
3
4
5
Perfectly
22
Q

How parvovirus B19 infects? Which cells?

A

RBC precursors in bone marrow ▶️ group P antigen (globoside) ▶️ receptor for B19

How well did you know this?
1
Not at all
2
3
4
5
Perfectly
23
Q

Acute and chronic sequelae of parvovirus B19 infection, who are most affected?

A
  • Chronic anemia ▶️ immunosupressed

- Aplastic crisis ▶️ underlying hematologic abnormality (sickle cell disease)

How well did you know this?
1
Not at all
2
3
4
5
Perfectly
24
Q

Most common malignancies associated with asbestos exposure.

A

Bronchogenic carcinoma>Malignant mesothelioma

How well did you know this?
1
Not at all
2
3
4
5
Perfectly
25
Q

What can explain new-onset odynophagia in context of chronic GERD disease?

A

Erosive esophagitis ▶️ ulcer formation

How well did you know this?
1
Not at all
2
3
4
5
Perfectly
26
Q

Which coagulation factor is tested by prothrombin time and no by activated partial thromboplastin time?

*PT ⬆️ and PTT normal

A

Factor VII (extrinsic pathway), also XIII.

*When do you think in factor VII deficiency?

How well did you know this?
1
Not at all
2
3
4
5
Perfectly
27
Q

Most common congenital heart disease in Turner syndrome

A

Bicuspid aortic valve disease (20-30%)

*Coarctation of the aorta (3-10%), aortic root dilation with ↑risk of aortic dissection in adulthood

How well did you know this?
1
Not at all
2
3
4
5
Perfectly
28
Q

Most common cardiac anomaly of Down Syndrome

A

Complete atrioventricular canal ▶️ atrial septal defect + ventricular septal defect

How well did you know this?
1
Not at all
2
3
4
5
Perfectly
29
Q

Common cardiac anomaly in marfan and ehlers-danlos syndrome and clinical finding.

A

Mitral valve prolapse

Mid-systolic click, late systolic murmur

How well did you know this?
1
Not at all
2
3
4
5
Perfectly
30
Q

In which cells there is an increase of what aminoacid during hepatic encephalopathy? Why is it accumulated?

A

Hyperammonemia
- NH3 taken by astrocytes ▶️ ⬆️ NH3 + glutamate ▶️ ⬆️ glutamine ▶️ ⬆️ intracellular osmolarity ▶️ swelling ▶️ ❌ glutamine release ▶️ no glutamine in neurons to make glutamate ▶️ 🚫 of excitatory neurotransmission.

How well did you know this?
1
Not at all
2
3
4
5
Perfectly
31
Q

Which compounds are major contributors to the progression of insulin resistance? How does it happen?

A

Insulin resistance ▶️ in adipose tissue ▶️ no antilipolytic effect ▶️ lipolysis:
Chronically ⬆️ fatty acids ▶️ ❌ insulin-dependent glucose uptake, ⬆️ hepatic gluconeogenesis

How well did you know this?
1
Not at all
2
3
4
5
Perfectly
32
Q

Ultrastructural change that suggest irreversible ischemic myocardial cell injury.

A

Mitochondrial vacuolization ▶️ unable to produce ATP

*Simple swelling ▶️ reversible cell injury

How well did you know this?
1
Not at all
2
3
4
5
Perfectly
33
Q

Cause of Duchenne dystrophy and what leads that fact? Why can you find elevated creatin kinase? Important sign.

A
  • Frameshift or nonsense mutation of dystrophin gene on X chromosome p21 ▶️ truncated protein ▶️ myonecrosis
  • Dystrophin prevents degradation of glycoproteins complexes at plasma membrane ▶️ if 🚫 ▶️ breakdown of glycoproteins and membrane ▶️ muscle fiber degeneration
  • Gower’s sign
How well did you know this?
1
Not at all
2
3
4
5
Perfectly
34
Q

Pathology finding in a Duchenne muscle dystrophy, to what sign agree?

A

Fibrofatty (fat and connective tissue) muscle replacement ▶️ pseudohypertrophy (common calf muscles)

How well did you know this?
1
Not at all
2
3
4
5
Perfectly
35
Q

Cause of humoral hypercalcemia of malignancy and mechanism. Most common tumors associated.

A
  • Secretion of Parathyroid hormone-related protein (PTHrP) ▶️ ⬆️ bone resorption ⬇️ renal excretion of Calcium ⏩ Hypercalcemia with NO focal lytic lesions.
  • Tumors secreting: squamous cell tumors (lung, neck, esophagus), renal, bladder, ovarian, breast, etc.

*PTH levels low - hypercalcemia suppression feedback

How well did you know this?
1
Not at all
2
3
4
5
Perfectly
36
Q

Causes and regions of watershed infarcts in the brain. How do they look in gross specimen?

A
  • Cardiogenic shock ▶️ profound hypotension ▶️ hypoxic-ischemic encephalopathy
  • Areas supplied by most distal branches of cerebral arteries ▶️ Border between areas perfused by ACA, MCA, PCA
  • Bilateral wedge-shaped strips of necrosis adjacent to the longitudinal cerebral fissure
How well did you know this?
1
Not at all
2
3
4
5
Perfectly
37
Q

Most probably histologic finding in women with blood discharge from the right nipple, no pain no masses or skin changes.

A

Intraductal papilloma (most common cause of alarming nipple discharge) ▶️ proliferation of papillary cells in a duct or cyst wall, with fibrovascular core ▶️ twist of vascular stalk ▶️ bleed

*may contain foci of atypia or ductal carcinoma in situ

How well did you know this?
1
Not at all
2
3
4
5
Perfectly
38
Q

Conditions that increase the risk of pigment gallstones. Why does it occur?

A
  • ⬆️ unconjugated bilirrubin ▶️ ⬆️ calcium bilirrubinate precipitation
  • Black stones ▶️ chronic hemolysis (sickle cell disease, B-thalasemia, spherocytosis), ⬆️ enterohepatic cycling of bilirrubin (ileal disease)
  • Brown stones ▶️ biliary tract infection (E. coli, liver fluke [Clonorchis sinensis]) ▶️ ⬆️ B-glucuronidase (release from injured hepatocytes and bacteria)
How well did you know this?
1
Not at all
2
3
4
5
Perfectly
39
Q

What allow distinguish silicosis of other pneumoconioses? Exposure associated.

A
  • Calcification of the rim of hilar nodes ▶️ eggshell calcification (chest x-ray)
  • Birefringent silica particles surrounded by fibrous tissue [dense collagen fibers (histology polarized microscopy)
  • Foundries, sandblasting, mines
How well did you know this?
1
Not at all
2
3
4
5
Perfectly
40
Q

Most common malignant hepatic lesion. Why?

A
  • Metastatic liver disease from another primary site (breast, lung, colon) ▶️ 20 times more common than hepatocellular carcinoma
  • Large size, dual blood supply, high perfusion rate, filtration function of Kuppfer cells ▶️ 2nd most common site of spread (after lymph nodes)
How well did you know this?
1
Not at all
2
3
4
5
Perfectly
41
Q

How do you diagnose erythroblastosis fetalis antenatally?

A

Amniocentesis ▶️ amniotic fluid bilirrubin levels ▶️ assess fetal hemolysis

How well did you know this?
1
Not at all
2
3
4
5
Perfectly
42
Q

How do you diagnose antenatally cystic fibrosis?

A

Amniocentesis ▶️fetal cells ▶️genetic mutation analysis▶️CFRT gene on chromosome 7, deletion Phe508

How well did you know this?
1
Not at all
2
3
4
5
Perfectly
43
Q

How do you diagnose adrenal fetal disorder antenatally? Example.

A
  • Amniocentesis ▶️ 17-hydroxyprogesterone levels, gene linkage analysis (HLA region of chromosome 6), abnormal gene with molecular probes
  • Congenital adrenal hyperplasia
How well did you know this?
1
Not at all
2
3
4
5
Perfectly
44
Q

Triad of Huntington disease

A

Movement disorder (Chorea), behavioral abnormalities (depression, apathy, aggressiveness), dementia.

How well did you know this?
1
Not at all
2
3
4
5
Perfectly
45
Q

Treatment of Crigler-Najjar syndrome

A
  • Type I ▶️ plasmaferesis and phototherapy

- Type II ▶️ less aggressive ▶️ fenobarbital ▶️ ⬆️ liver enzymes synthesis

How well did you know this?
1
Not at all
2
3
4
5
Perfectly
46
Q

Disease which increases unconjugated bilirrubin until certain point that elevates it in brain, and deposits on it, leading in kernicterus. What is the cause?

A

Crigler-Najjar syndrome ▶️ Uridine Glucuronyltransferase (UGT) deficiency

How well did you know this?
1
Not at all
2
3
4
5
Perfectly
47
Q

How do you identify a 11B-hydroxylase deficiency?

A
  • Female virilization
  • No cortisol and aldosterone ▶️ precursors diverted toward androgens synthesis; ⬆️ ACTH ▶️ ⬆️ androgens
  • Low-renin HTN and hypokalemia: ⬆️ 11-deoxycorticosterone (weak mineralocorticoid)
How well did you know this?
1
Not at all
2
3
4
5
Perfectly
48
Q

Why precocious puberty could lead in a shorter stature despite initial growth spurt?

A

⬆️ sex steroid hormones ▶️ ⬆️ linear growth initially but also closure of long bone epiphyseal plate (“growth plate”)

How well did you know this?
1
Not at all
2
3
4
5
Perfectly
49
Q

Hallmarks and red flags that suggest an abusive head trauma in a infant (shaken baby syndrome).

A

Lethargy, Retinal hemorrhages (highly suggestive), subdural hemorrhages acute or chronic, posterior rib fractures, inconsistency with history or developmental age

How well did you know this?
1
Not at all
2
3
4
5
Perfectly
50
Q

When occur the formation of cholesterol gallstones?

A

Water insoluble cholesterol ▶️ “made” soluble by bile acids (also from cholesterol) and phosphatidylcholine ▶️ when ⬆️ cholesterol>bile acids ⬇️, phosphatidylcholyne ▶️ more cholesterol to make soluble ▶️ precipitates into crystals ▶️ gallstones

How well did you know this?
1
Not at all
2
3
4
5
Perfectly
51
Q

Identify by clinic Ehler-danlos syndrome, stage in collagen synthesis that is disrupted.

A
  • Easy bruising, hyperextensible skin, hypermobile joints, poor wound healing
  • Procollagen peptidase deficiency ▶️ No proteolytic processing in EC matrix ▶️ no cleavage of disulfide-rich terminal regions of procollagen ▶️ soluble tropocollagen can’t crosslink
How well did you know this?
1
Not at all
2
3
4
5
Perfectly
52
Q

What drug do you use to confirm diagnosis of asthma during spirometry, and when do you use it?

A

Normal spirometry, and history suggest asthma ▶️ methacoline ▶️ muscarinic cholinergic agonist ▶️ bronchoconstriction and ⬆️ mucus production ▶️ broncoprovocation ⏩ ⬇️ FEV1 after mechacoline challenge

How well did you know this?
1
Not at all
2
3
4
5
Perfectly
53
Q

Usual findings in a asthma patient spirometry

A

⬇️ FEV1

⬇️ Peak expiratory flow rate

How well did you know this?
1
Not at all
2
3
4
5
Perfectly
54
Q

What is the mechanism and findings of Familial Hypocalciuric Hypercalcemia (FHH)?

A
  • Mutation in CaSR (calcium sensing receptors) = G-protein coupled receptor in renal and parathyroid gland ▶️ sense blood calcium ▶️ control secretion of PTH
  • Higher Ca levels required to suppress PTH secretion
  • Mild asymptomatic hypercalcemia, ⬇️ excretion urinary calcium (excessive renal Ca reuptake), mildly elevated PTH
How well did you know this?
1
Not at all
2
3
4
5
Perfectly
55
Q

What is the use of Sudan III stain for stool? How does it work?

A
  • Most sensitive strategy for screen when suspiction of Generalized malabsorption disorder
  • Look for fat (normally unmeasurable) in stool→Fat is the main macronutrient affected in malabsorption
How well did you know this?
1
Not at all
2
3
4
5
Perfectly
56
Q

When do you suspect Allergic Bronchopulmonary Aspergillosis (ABPA)? Diagnose.

A
  • History of cyistic fibrosis or asthma (usually corticoid-dependent - 5-10%) + recurrent fleeting infiltratres and Bronchiecatasis.
  • ↑IgE, (+) skin test, serum IgG for Aspergillus, eosinophilia
How well did you know this?
1
Not at all
2
3
4
5
Perfectly
57
Q

High risk of hematologic malignancy in Down syndrome?

A
  • 10 to 20 fold increase Acute lymphoblastic leukemia

- Acute myelogenous leukemia

How well did you know this?
1
Not at all
2
3
4
5
Perfectly
58
Q

What do you see in a karyotype of Chronic myelogenous leukemia? Why does it happen?

A
  • Elongation of chromosome 9
  • Reciprocal translocations between the long arms of chromosomes 9 (ABL gene) and 22 (BCR gene) 16►BCR-ABL oncogenic fusion gene on shortened chromosome 22→Philadelphia chromosome
How well did you know this?
1
Not at all
2
3
4
5
Perfectly
59
Q

Serum sickness histology finding. Causes and mechanism.

A
  • Fibrinoid necrosis, neutrophils infiltration in small blood vessels ▶️ vasculitis
  • Type III hypersensitivity ▶️ deposition IgG and/or IgM - Ag complexes ▶️ activating complement ▶️ consum complement ▶️ ⬇️ C3
  • Antigenic heterologous proteins ▶️ chymeric monoclonal antibodies (infliximab, rituximab), non-human proteins (venom antitoxin)
How well did you know this?
1
Not at all
2
3
4
5
Perfectly
60
Q

Most common secondary neoplasia in a patient with familial retinoblastoma

A

Sarcomas (Osteosarcoma most common)

How well did you know this?
1
Not at all
2
3
4
5
Perfectly
61
Q

What can result when rapid lysis (chemotherapy) of leukemic cells of Acute myelogenous leukemia (AML) type M3 occur, and why?

A

Auer Rods in cytoplasm (enzymes that are found in azurophilic granules)→↑ release acid phosphatase, peroxidases, esterases→ (+) Disseminated Intravascular Coagulation (DIC)

How well did you know this?
1
Not at all
2
3
4
5
Perfectly
62
Q

Treatment of AML type M3 to improve outcome and prevent initiation of DIC.

A

All-trans retinoic acid (ATRA)→(+)RARα (retinoic acid alpha receptor)→differentiation in hematopoietic cells→mature neutrophils

How well did you know this?
1
Not at all
2
3
4
5
Perfectly
63
Q

Diseases associated with secondary amyloidosis, which type of protein is accumuladted and where?

A
  • Autoimmune diseases or Chronic inflammatory conditions (rheumatoid arthritis, IBD, spondyloarthropathy, familial Mediterranean fever), chronic infection.
  • AA amyloid type→systemic kidney, liver, heart, spleen; most common presentation→renal insuficiency/nephrotic syndrome
How well did you know this?
1
Not at all
2
3
4
5
Perfectly
64
Q

Paraneoplasic cause of SIADH secretion. Clinical findings.

A
  • Bronchogenic small cell carcinoma

- confusion, weakness, fatigue, weight gain ▶️ dilutional hyponatremia

How well did you know this?
1
Not at all
2
3
4
5
Perfectly
65
Q

Which is the most important determinant of severity of symptoms (degree of cyanosis) and prognosis in tetralogy of fallot?

A

Pulmonary infundibular stenosis or right ventricular outflow tract obstruction

How well did you know this?
1
Not at all
2
3
4
5
Perfectly
66
Q

What laboratory finding (blood smear) do you expect in a patient that has a mutation leading decrease ß-globin chain synthesis? what diagnosis?

A
  • ß-thalassemia→small RBC ↑ central pallor
  • Microcytosis, hypochromic RBC, basophilic stippling, anisopoikilocytosis, target cells, teardrop cells, nucleated RBC precursors.
How well did you know this?
1
Not at all
2
3
4
5
Perfectly
67
Q

Common association of hyperchromy in blood smear. Cause of that disease.

A
  • Hereditary spherocytosis

- Variety of defects in genes that code for RBC membrane proteins (ankyrin, spectrin, band 3, protein 4.2)

How well did you know this?
1
Not at all
2
3
4
5
Perfectly
68
Q

Which diagnose do you suspect in a patient with recurrent intranasal uclers, recent onset renal failure? Which antibodies are associated? Pathologic findings.

A
  • Wegener’s granulomatosis (with polyangiitis)→C-ANCAS (target neutrophil proteinase 3)
  • Necrotizing vasculitis of the upper and lower respiratory tract (nasal ulcers, sinusitis, hemoptysis) and rapidly progressive glomerulonephritis
How well did you know this?
1
Not at all
2
3
4
5
Perfectly
69
Q

Cause of hypercalcemia in a granulomatous disease? What others diseases show same mechanism and hypercalcemia?

A
  • Sarcoidosis, TB→PTH independent coversion to 1,25 OH Vitamin D→↑Ca intestinal absorption→hypercalcemia and hypercalciuria
  • Expression of ­1-alpha-hydroxylase in activated macrophages
  • Hodgkin disease and non-hodgkin lymphoma
How well did you know this?
1
Not at all
2
3
4
5
Perfectly
70
Q

What suggests an elderly patient with nausea, vomiting, mild abdominal pain, CT abdomen scan ileal wall thickening and no enhancement with contrast? Which enzyme would be decreased in activity if patient have lactic acidosis?

A
  • Acute mesenteric ischemia
  • Hypoxic enviroment→accumulation of NADH→(-)Pyruvate dehydrogenase (↓ activity)→↑LDH activity►Pyr→Lactate►lactic acidosis
How well did you know this?
1
Not at all
2
3
4
5
Perfectly
71
Q

What mutations are involved in Lynch syndrome and what is the consequence in processing the DNA?

A
  • MSH2, MLH1 (MutS, MutL proteins - 90% of lynch syndrome cases) ▶️ Missmatch repair
  • Defective DNA Missmatch repair
How well did you know this?
1
Not at all
2
3
4
5
Perfectly
72
Q

What do you suspect in a patient that first presents deep vein thrombosis and then middle cerebral artery stroke? How does it happen? Most common causes.

A
  • Paradoxicall embolism ▶️ emboli from systemic venous circulation crosses to systemic arterial circulation through intracardiac or intrapulmonary shunt.
  • ASD, patent forman ovale, VSD, large pulmonary arteriovenous malformation
  • Transient reversal of the shunt ▶️ left-right shunt condition, become right-left shunt during periods of ⬆️ right side pressure (early ventricular systole, cough, defecation)
How well did you know this?
1
Not at all
2
3
4
5
Perfectly
73
Q

Auscultation findings in atrial septal defect.

A
  • Wide and fixed splitting of S2 (no change with respiration) ▶️ equalization of the left and right atrial pressures
  • Mid systolic murmur ▶️ ⬆️ flow through pulmonary valve
  • Mid diastolic rumble ▶️ ⬆️ flow through tricuspid valve
How well did you know this?
1
Not at all
2
3
4
5
Perfectly
74
Q

Which finding in the auscultation do you expect to find in VSD?

A
  • VSD ▶️ most common congenital heart disease (25% of all cases)
  • Holosystolic murmur in left sternal border at 3rd or 4th intercostal space
How well did you know this?
1
Not at all
2
3
4
5
Perfectly
75
Q

Main features of dysplasia

A
  • Referred only to epithelial cells
  • Reversible
  • Pleomorphism (differing in shape and size)
  • Hyperchromatism
  • Disorganized (loss alignment along basement membrane)
  • ⬆️ mitoses
  • N/C ratio may be ⬆️ (more in carcinoma in situ)
  • High-grade dysplasia (entire epithelium) = carcinoma in situ
  • Low-grade dysplasia not involve entire epithelium
How well did you know this?
1
Not at all
2
3
4
5
Perfectly
76
Q

Functions and location of parietal cells.

A
  • Produce: intrinsic factor→absorption of vitamin B12; gastric acid
  • Oxyntic, round, plate-like, in upper glandular layer
How well did you know this?
1
Not at all
2
3
4
5
Perfectly
77
Q

Congenital bicuspid aortic valve is associated with what valve disease at 50’s?

A

Aortic stenosis >50% of afected→accelerated aging process→premature atherosclerosis and calcification

How well did you know this?
1
Not at all
2
3
4
5
Perfectly
78
Q

What cells are implicated in centriacinar emphysema and why?

A
  • Neutrophils and Macrophages→elastase, proteases, cathepsines, etc→degrade extracellular matrix; ↑O2 free radicals→inactivated protease inhibitors (alpha1-antitrypsin)►acinar wall destruction (irreversible airspace dilation distal to the terminal bronchioles)
  • alpha1-antitrypsin deficiency→panacinar emphysema
How well did you know this?
1
Not at all
2
3
4
5
Perfectly
79
Q

What do you suspect if find gastric varices alone at esophagus and rest of stomach normal, which vessel would have increase pressure and why?

A
  • Splenic vein thrombosis ▶️ ⬆️ pressure inside it ▶️ short gastric veins of fundus drain on it ▶️ dilate ▶️ gastric varices
  • Splenic vein courses posterior of pancreas ▶️ inflammatory process of pancreas (chronic pancreatitis, cancer) ▶️ clot formation

*gastric varices also seen in portal hypertension (⬆️ pressure in left gastric veins) but with esophageal varices

How well did you know this?
1
Not at all
2
3
4
5
Perfectly
80
Q

Which parameter is a measure of visceral fat and an indicator of high risk of insulin resistance, diabetes mellitus and coronary heart disease?

A

Waist circumference or waist-to-hip ratio
>102 cm (40 inch) in male
>88 cm (35 inch) in women

How well did you know this?
1
Not at all
2
3
4
5
Perfectly
81
Q

What vasculitis have a common pathological finding, characterized by granulomatous inflammation of the media? How do you differentiate them?

A
  • Takayasu arteritis ▶️ aorta and proximal aortic arch branch, female <40
  • Temporal arteritis (Giant cell arteritis - GCA) ▶️ more distal carotid artery branch, patients >50

*Different clinical presentation. Jaw claudication specific for GCA

How well did you know this?
1
Not at all
2
3
4
5
Perfectly
82
Q

What suggest to you a culture of immature cells that differentiate when are exposed to vitamin A in a elderly patient with DIC? Why?

A
  • Acute myelogenous leukemia (Acute promyelocytic leukemia - APL), type M3 ▶️ t(15;17) mutation ▶️ fusion of retinoic acid receptor alpha gene with promyelocytic leukemia gene ▶️ PML/RARɑ ▶️ unable to signal the differentiation
  • normally RARɑ ▶️ proper differentiation of myeloid precursors when bind Retinoic acid ▶️ affect genes for maturation. Tx: all-trans-retinoic acid (ATRA) ▶️ partly induce PML/RARɑ proteolysis, (+) differentiation of myeloblasts (remission in 90%)
How well did you know this?
1
Not at all
2
3
4
5
Perfectly
83
Q

What condition can result when there is a decreased number of antral cells that produce somatostatin? How does it occur and what is the cause?

A
  • Duodenal ulceration (duodenal gastric metaplasia)
  • Helicobacter pylori (damage gastric mucosa by localized inflammation - antral gastritis) ▶️ ⬇️ antral cells somatostatin producers (delta cells) ⏩ ⬆️ gastrin ▶️ ⬆️ H+ (parietal cells)

*somatostatin normally ▶️⬇️ gastrin (antral G cells)▶️ ⬇️ H+ (parietal cells)

How well did you know this?
1
Not at all
2
3
4
5
Perfectly
84
Q

Neoplasia associated with myasthenia gravis? Which embryologic origin has the tissue of the neoplasia, what other structures come from that origin?

A
  • Thymoma o thymic hyperplasia

- 3dr pharyngeal pouch→thymus, inferior parathyroid gland

How well did you know this?
1
Not at all
2
3
4
5
Perfectly
85
Q

Why chronic kidney disease can lead in secondary hyperparathyroidism?

A
  • CKD ▶️ ❌ 25-hydroxyvitamin D ▶️ 1,25 dihydroxyvitamin D (active form) by 1-alpha hydroxylase. ❌ glomerular and tubular function ⏩ hyperphosphatemia, hypocalcemia, ⬇️ vitamin D ▶️ ⬆️ PTH
  • weakness, bone, muscle pain, abnormal bone mineralizarion, ⬆️ risk of fracture.
How well did you know this?
1
Not at all
2
3
4
5
Perfectly
86
Q

What suggest and what is a finding of pneumatosis intestinalis in a premature infant with bloody stools and abdominal distention? Why does it occur?

A
  • on X-ray ▶️ air in the bowel wall ▶️ thin curvilinear lucency parallel to the lumen
  • Necrotizing enterocolitis
  • Immature immune clearance ▶️ initiate enteral feeding ▶️ get in bacterias ▶️ ⬆️ proliferation ▶️ invade bowel wall ▶️ inflammation and ischemic necrosis ▶️ intramural gas collection
How well did you know this?
1
Not at all
2
3
4
5
Perfectly
87
Q

What neurotransmitter is changed in alzheimer disease, why?

A
  • Ach in basal nucleus of Meynert (memory and cognition) and hippocampus (new memories formation)
  • Choline acetyltransferase deficiency
How well did you know this?
1
Not at all
2
3
4
5
Perfectly
88
Q

Cause of hereditary pancreatitis

A
  • Mutations in trypsin gene ▶️ less sensitive trypsin to be cleaved by itself
  • Mutations in serine peptIdase inhibitor Kazal type 1 (SPINK1) ▶️ trypsin inhibitor ▶️ avoid activation of trypsinogen prematurely within pancreas (autodigestion)

*premature activation of trypsinogen before it reaches the duodenal lumen ▶️ autodigestion of pancreas

How well did you know this?
1
Not at all
2
3
4
5
Perfectly
89
Q

When do you suspect an amniotic fluid embolism? What is the histologic finding?

A
  • During pregnancy or short after delivery ▶️ amniotic fluid [arachidonic acid, tissue factor (thromboplastin)] enter maternal circulation ▶️ uterine trauma sites, cervical lacerations ▶️ anaphylactoid reaction ⏩ occlusion and vasospasm of maternal pulmonary circulation ▶️ left ventricular failure, ⬇️ cardiac output, seve V/Q mismatch ▶️ hypoxia, hypotensive shock ▶️ cardiopulmonary arrest
  • Fetal squamous cells and mucin in maternal pulmonary arteries
How well did you know this?
1
Not at all
2
3
4
5
Perfectly
90
Q

Classic histologic findings of Burkitt lymphoma. Organism associated with developing of the tumor and why is it related?

A
  • Sheets of diffuse-medium size lymphocyte, high proliferation index (high Ki-67 fraction approaching to 100%), “starry sky” appereance→interspersed “body tingible” macrophages
  • EBV→↑risk of c-myc translocation t(8;14)→overexpression of c-myc→nuclear phosphoprotein transcriptional regulator (activator)→control cell proliferation
How well did you know this?
1
Not at all
2
3
4
5
Perfectly
91
Q

What is the müllerian aplasia and what study you should make because highly associated anatomical alteration?

A
  • Defective development or congenital absence of müllerian ducts→variable uterine development, no upper vagina►normal ovaries→regular development of secondary sexual characteristics (breast, pubic hair), primary amenorrhea
  • Renal ultrasound→50% will have a coexisting urologic anomaly (unilateral renal agenesis)
How well did you know this?
1
Not at all
2
3
4
5
Perfectly
92
Q

What condition do you suspect if a young patient has acne, testicular atrophia (in case of female virilization→clitoromegaly, hirsutism), increased hematocrit?

A

Androgenic steroid abuse

How well did you know this?
1
Not at all
2
3
4
5
Perfectly
93
Q

CT findings on constrictive pericarditis and possible causes. Common clinical presentation.

A
  • Thickening and calcification of the pericardium (normally 1-2 mm thick, constrictive pericarditis 4-20 mm)
  • Radiation therapy to the chest, cardiac surgery, TB
  • Slowly progressive dyspnea, chronic edema, ascitis
How well did you know this?
1
Not at all
2
3
4
5
Perfectly
94
Q

What change do you expect to find in a brain after more than 1 month of ischemic stroke? what cells mediate that change?

A

Cystic area sorrounded by dense glial fibers→astrocytes forming glial scar previously (2 wks after stroke)

How well did you know this?
1
Not at all
2
3
4
5
Perfectly
95
Q

What is the most common mutation of the CFTR gene and what is the consequence?

A

CFTR gene→ATP-gated Cl− channel►secretes Cl− in lungs and GI tract, reabsorbs Cl− in sweat glands
- F508 mutation→Misfolded protein (abnormal postranslational processing)→detected in RER→target for proteasome degradation→not transported to cell membrane→↓Cl− (and H2O) secretion►↑intracellular Cl−→compensatory ↑Na+, H20 reabsorption→thick mucus (bronchioles, pancreatic ducts, bile
ducts, meconium, cervix, seminal fluid)
- ↓Reabsorption of Cl in sweat glands→↑Cl− concentration (> 60 mEq/L) in sweat (Dx)

*Na+ and water follow Cl-→CFTR hydrates mucosal surfaces in the airways and bowel

How well did you know this?
1
Not at all
2
3
4
5
Perfectly
96
Q

Why do the patients with cystic fibrosis have elevated chloride in the sweat?

A

Normally NaCl is removed from the ductal lumen of the eccrine sweat gland→transform isotonic to hypotonic sweat

*Absence of functional CFTR→cannot remove salt from the sweat→↑sweat Cl- (also Na+)

How well did you know this?
1
Not at all
2
3
4
5
Perfectly
97
Q

What suggest to you the presence of schistocytes in a blood smear? Possible causes of that mechanism.

A

♦Hemolytic anemia by traumatic mechanism: RBC damaged when passing through obstructed or narrowed vessel lumina
- Microangiopathic hemolytic anemia→Hemolytic uremic syndrome (HUS), thrombotic thrombocytopenic purpura (TTP), CID, malignant hypertension, metastatic carcinoma
♦Hemolytic anemia by mechanical damage:
- Macroangiopathic hemolytic anemia→Prosthetic valve, severely calcified valves (aortic stenosis)

  • HUS and TPP→normal coagulation studies
  • CID→abnormal coagulation studies
How well did you know this?
1
Not at all
2
3
4
5
Perfectly
98
Q

Bloody diarrhea a week ago, schistocytes of blood smear and hemolytic anemia, thrombocytopenia, also acute renal failure. Most probably diagnosis and possible cause.

A
  • Hemolytic uremic syndrome (HUS)

- Shiga-like toxin→enterohemorrhagic E. coli (O157:H7) or shigella

How well did you know this?
1
Not at all
2
3
4
5
Perfectly
99
Q

Why are the patients with chron disease prone to develop gallstones?

A

Inflammation at terminal ileum→no enough recycling bile acids→↑bile acids wasting→↑ratio cholesterol/bile acid→precipitation of cholesterol in bile→ gallstones

How well did you know this?
1
Not at all
2
3
4
5
Perfectly
100
Q

How do you expect to find the serum markers of a exacerbation (infective) of chronic hepatitis B?

A
  • “Healthy” chronic carrier: HBsAg, anti-HBc IgG→contagious but at a much ↓risk
  • Infective chronic carrier: HBsAg, anti-HBc IgG, infective particles (↑HBV DNA, HBeAg), Abrupt ↑ liver enzymes (exacerbation - acute hepatitis)→↑risk for postnecrotic cirrhosis and hepatocellular carcinoma
How well did you know this?
1
Not at all
2
3
4
5
Perfectly
101
Q

Most common group of frequency of intussusception, cause associated, representative features in its clinic presentation and most common place of occurence at the GI tract.

A
  • <2 years old children (often without any structural cause, sometimes associated with viral infection)
  • In >2 years old→meckel diverticulum, foreign body, intestinal tumor
  • “Currant jelly” stools (contain mucus and blood), tubular palpable mass at lower right quadrant
  • Ileocecal valve→size differents in adjacents segments→small bowel invaginate into the cecum
How well did you know this?
1
Not at all
2
3
4
5
Perfectly
102
Q

Diagnostic and treatment of intussusception

A

Barium enema→Dx and Tx, if not resolve with it→surgery

How well did you know this?
1
Not at all
2
3
4
5
Perfectly
103
Q

Composition of crescents in the kidney. What diseases usually show this finding?

A
  • Abundant Fibrin + plasma proteins (IgG, C3)
  • Macrophages, monocytes, glomerural parietal cells
  • Rapidly progressive glomerulonephritis (RPGN), Goodpasture syndrome (type 1 RPGN), granulomatosis with polyangiitis, microscopic polyangiitis, etc
How well did you know this?
1
Not at all
2
3
4
5
Perfectly
104
Q

How do you distinguish hypercalcemia caused by thiazide diuretic side effect and hyperparathyroidism?

A

PTH level

  • ↓ in thiazide diuretic side effect
  • ↑ in Hyperparathyroidism
How well did you know this?
1
Not at all
2
3
4
5
Perfectly
105
Q

Typical signs and symptoms of thrombotic thromocytopenic purpura (TTP) and mechanism of disease

A
  • New onset neurologic symptoms, anemia with schistocytes, thrombocytopenia, acute kidney injury, fever, normal PT/PTT, ↑bleeding time, ↑LDH, ↓haptoglobin, cutaneous petechiae, purpuric lesions
  • ↓ADAMTS13 (vWF cleaving protease) activity→large vWF multimers→diffuse microvascular platelet rich thrombi

*Acquired (antibodies) or hereditary

How well did you know this?
1
Not at all
2
3
4
5
Perfectly
106
Q

When do you think in polyartheritis nodosa? and with what organism is often associated?

A
  • Multisystemic organ involvement (most common kidney, heart, liver, GI tract, muscles)→symptoms will be due ischemia of the involved organs
  • Bx→transmural inflammation of the arterial wall with fibrinoid necrosis
  • Hepatitis B associated in 30% of cases
How well did you know this?
1
Not at all
2
3
4
5
Perfectly
107
Q

Why pheochromocytoma causes hypokalemia? Difference from hyperaldosteronism (also HTN and hypokalemia).

A
  • ⬆️ Epinephrine ▶️ B2 receptor ▶️ intracellular K+ shift

- Unlike hyperaldosteronism, pheochromocytoma ▶️ tachycardia, increased sweating.

How well did you know this?
1
Not at all
2
3
4
5
Perfectly
108
Q

Differential diagnosis of patient with hypokalemia, hypertension, ⬆️ renin, ⬆️ aldosterone and muscle weakness. Cause of each one.

A

Secondary hyperaldosteronism

  • Renovascular HTN ▶️ Renal artery stenosis (fibromuscular dysplasia or atherosclerosis)
  • Diuretic use
  • Malignant HTN ▶️ microvascular damage and renal ischemia
  • Renin secreting tumors (Reninoma) ▶️ Juxtaglomerual cell tumor
How well did you know this?
1
Not at all
2
3
4
5
Perfectly
109
Q

Lesion in what hypothalamic nuclei can result in hyperphagia and obesity

A

Ventromedial nucleus ▶️ control satiety and monitor blood glucose levels

How well did you know this?
1
Not at all
2
3
4
5
Perfectly
110
Q

How do you identify a Paget disease of bone and which cell type predominates in each of its phases?

A
  • Bone pain, tenderness point, ↑phosphatase alkaline, elderly (until here also bone metastases), Bx→haphazardly oriented segments of lamellar bone with prominent cement lines (mosaic pattern)
    1. Osteolytic→osteoclast dominant phase
    2. Mixed (osteoclastic/osteoblastic)→↑bone formation with persistent resorption
    3. Osteosclerotic→osteoblast dominant phase→continued bone formation

*Thick and deformed bone weaker than normal and prone to fracture

How well did you know this?
1
Not at all
2
3
4
5
Perfectly
111
Q

What cardiovascular alteration can result in Paget bone disease and why?

A
  • High-output heart failure
  • Osteoclasts→↑cytokines→↑proliferation of endothelial cells and fibroblasts→adjacent marrow spaces replaced by highly vascular stromal tissue (↑vascularity)►arteriovenous shunting
How well did you know this?
1
Not at all
2
3
4
5
Perfectly
112
Q

What do you look for in a bone marrow aspirate in a patient with Gaucher disease? In what other tissues can you find them?

A
  • Lipid-laden macrophages (Gaucher cells)→”wrinkled tissue paper”, wrinkled silk”, “crumpled newspaper”
  • Liver, spleen, lymph nodes, tonsils, etc
How well did you know this?
1
Not at all
2
3
4
5
Perfectly
113
Q

Typical clinical presentation of Gaucher disease

A
  • Bone pain→bone marrow invasion/inflammation
  • Abdominal distension→Hepatosplenomegaly
  • Easy bruising, fatigue, pallor→Pacytopenia (more common anemia and thrombocytopenia than leukopenia)
How well did you know this?
1
Not at all
2
3
4
5
Perfectly
114
Q

Possible causes of pancytopenia without splenomegaly

A
  • Aplastic anemia
  • Severe B12 and folate deficiency anemia
  • Acute leukemias
  • Certain forms of myelodysplastic syndrome
How well did you know this?
1
Not at all
2
3
4
5
Perfectly
115
Q

Diagnose and treatment of vWF deficiency.

A
  • Dx→Ristocetin aggregation test→asses in vitro vWF dependent platelet aggregation►poor aggregation that correct when addition normal plasma (contain vWF)
  • Tx→combined oral contraceptives for menorrhagia, desmopression (↑release vWF from endothelium)
How well did you know this?
1
Not at all
2
3
4
5
Perfectly
116
Q

What is the response to Ristocetin aggregation test in Bernard-soulier syndrome? why?

A
  • Hereditary deficiency of GP Ib receptors→thrombocytopenia, enlarged platelets, mucocutaneous bleeding
  • Abnormal aggregation with Ristocetin, but normal plasma addition WON’T correct aggregation (vWF levels are normal, problem is GP Ib receptor)
How well did you know this?
1
Not at all
2
3
4
5
Perfectly
117
Q

Rrepresentative feature of congenital deficiency of factor XII (Hageman)

A
  • Marked PTT prolongation without bleeding diathesis

- Tendency for thromboembolic complications

How well did you know this?
1
Not at all
2
3
4
5
Perfectly
118
Q

Why do you see hemolysis in pyruvate kinase deficiency? and why do you find splenomegaly?

A
  • Pyruvate kinase deficiency→PEP►Pyruvate (glycolysis)→ATP→need to maintain RBC structure (ATP for transport cations against a concentration gradient in RBC membrane→disrupt gradient→H2O and K+ loss→defective maintain membrane architecture►echinocyte and hemolysis)
  • Spleen→red pulp hyperplasia by removing defective RBC
How well did you know this?
1
Not at all
2
3
4
5
Perfectly
119
Q

What findings can you see in a blood smear of a glucose 6P dehydrogenase deficiency patient?

A

Bite cells and Heinz bodies

How well did you know this?
1
Not at all
2
3
4
5
Perfectly
120
Q

Mechanism of disease of sickle cell disease and findings in blood smear.

A
  • Missense mutation in beta-globin chain→Hb S→Polymerization in deoxygenated states►RBC membrane injury and deformation (“sickle shape”)
  • Drepanocytes
How well did you know this?
1
Not at all
2
3
4
5
Perfectly
121
Q

Findings in the blood smear of Thalassemias

A
  • Target cells

- Hypochromic microcytes

How well did you know this?
1
Not at all
2
3
4
5
Perfectly
122
Q

Findings in the blood smear of megaloblastic anemia

A
  • Oval cells

- Hypersegmented neutrophils

How well did you know this?
1
Not at all
2
3
4
5
Perfectly
123
Q

Classic clinical manifestations and blood smear of Hereditary spherocytosis and mechanism of disease

A
  • Autosomal dominant mutation→spectrin and ankirin (plasma-membrane scaffolding proteins)→spherocytes less deformable→prone to sequestration and destruction by spleen
  • Hemolytic anemia, jaundice, splenomegaly
How well did you know this?
1
Not at all
2
3
4
5
Perfectly
124
Q

What is the product of c-Myc gene? in what translocation associated to lymphoma is involved and what type of lymphoma causes it? Histologic finding.

A
  • Nuclear phosphoprotein as a transcription activator→cell proliferation, differentiation, apoptosis
  • t(8;14)→Burkitt lymphoma
  • “Starry sky” appearance, monomorphic intermediate size lymphocytes with round nuclei, multiple prominent nucleoli, vacuolated basophilic cytoplasm
How well did you know this?
1
Not at all
2
3
4
5
Perfectly
125
Q

Translocation associated to follicular lymphoma and mantle lymphoma, and their consequences in the cell.

A
  • Follicular lymphoma→t(14;18)→BCL2 overexpression→apoptosis inhibitor
  • Mantle lymphoma→t(11;14)→cyclin D1 overexpression→promoter of G1►S-phase transition
How well did you know this?
1
Not at all
2
3
4
5
Perfectly
126
Q

Translocation of associated with chronic myelogenous leukemia and its consequence.

A
  • t(9;22) [philadelphia chromosome]→BCR-ABL (resulting fusion gene)→inhibits apoptosis, ↑mitogenesis, ↑tyrosine kinase activity
How well did you know this?
1
Not at all
2
3
4
5
Perfectly
127
Q

Causes and mechanism of communicating hydrocephalus and how do you find the ventricles?

A
  • Sequelae of meningeal infection (include TB meningitis) or subarachnoid/intraventricular hemorrhage→dysfunction or obliteration of subarachnoid villi►impair absoprtion of CSF
  • Increased CSF production→Ex, choroid plexus papilloma
  • All ventriculi symmetrical enlarged

*↑intracranial pressure, papilledema, herniation

How well did you know this?
1
Not at all
2
3
4
5
Perfectly
128
Q

Paradoxical brain embolism causes.

A
  • Persistent foramen ovale▶️incomplete fusion of atrial septum primum and secundum
  • Atrial septal defect▶️aplasia of the septum primum or septum secundum
How well did you know this?
1
Not at all
2
3
4
5
Perfectly
129
Q

How could be the clinical presentation of gestation choriocarcinoma and why? Histologic feature of it.

A
  • Dyspnea, hemoptisis▶️pulmonary metastasis [most common site] (hematogenous spread)
  • Anaplasic proliferation of cytotrophoblasts and syncytiotrophoblasts without villi

*Followed by normal pregnancy (usual) or molar, ectopic, aborted. Vagina bleeding, uterine enlargement, ⬆️⬆️ B-hCG

How well did you know this?
1
Not at all
2
3
4
5
Perfectly
130
Q

Risk factor for endometrial cancer. What factors can cause it?

A
  • Endometrial hyperplasia▶️crowded endometrial glands lined by atypical cells
  • Prolonged exposure to high estrogen levels: obesity, nulliparity, tamoxifen

*vaginal bleeding in a postmenopausal woman

How well did you know this?
1
Not at all
2
3
4
5
Perfectly
131
Q

What is the Stewart-Treves syndrome and why could it happen?

A
  • Cutaneous angiosarcoma resulting from chronic lymphedema (risk factor)
  • Ex: Radical mastectomy with axillary lymph node dissection. Any cause of chronic lymphedema.
How well did you know this?
1
Not at all
2
3
4
5
Perfectly
132
Q

Gas arterial disturbance in high-altitude sickness. Why does it happen?

A
  • Respiratory alkalosis compensated in 24-48 hrs by renal bicarbonate excretion
  • Hypobaric hypoxia▶️⬆️altitude▶️⬇️barometric pressure▶️⬇️pO2 in air and blood▶️hypoxia▶️➕peripheral chemoreceptors▶️hyperventilation (try to improve oxygenation)▶️⬇️pCO2
How well did you know this?
1
Not at all
2
3
4
5
Perfectly
133
Q

How do you explain the hypercalcemia in multiple myeloma? how are the levels of PTH, urinary calcium, vitamin D and PTHrp?

A
  • Osteolysis induced by tumor cells→↑osteoclasts activating factors (IL-1)
  • Myeloma cells→↑Inhibitor of osteoblasts differentiation (DKK1)
  • PTH↓, urinary calcium↑, vitamin D↓, PTHrp normal
How well did you know this?
1
Not at all
2
3
4
5
Perfectly
134
Q

How do you explain hypercalcemia in granulomatous disorder?

A

Expression of alpha-1-hydroxylase→↑1,25 dihydroxyvitamin D→↑intestinal absorption of calcium→↑blood calcium→↓PTH, hypercalciuria

*Sarcoidosis, Tuberculosis

How well did you know this?
1
Not at all
2
3
4
5
Perfectly
135
Q

What hematologic disturbances can you find in systemic lupus erythematosus and how do they occur?

A

Type II hypersensitivity reaction:

  • Anemia→warm IgG antibodies against RBC→autoimmune hemolysis►spherocytosis, positive direct coombs test, extravascular hemolysis
  • Thrombocytopenia→antibodies against platelets (as idiopathic thrombocytopenic purpura)
  • Leukopenia→antibodies against neutrophils
How well did you know this?
1
Not at all
2
3
4
5
Perfectly
136
Q

In a mitral stenosis what finding about pressures and volumes at the heart suggest a concomitant aortic valve defect? Probably cause of it.

A

⬆️left ventricular diastolic pressure▶️aortic stenosis and/or regurgitation⏩rheumatic heart disease

*Usually in mitral stenosis alone ▶️⬇️or normal left ventricular diastolic pressure

How well did you know this?
1
Not at all
2
3
4
5
Perfectly
137
Q

What is the cause and mechanism of CID in a pregnant woman?

A
  • Placental injury▶️placental trophoblast▶️Tissue factor (thromboplastin)▶️➕coagulation cascade⏩circulating microthrombi▶️organ ischemia, fibrinolysis (⬆️D-dimer) and coagulation factors rapidly consumed (⬇️platelets, ⬆️PT PTT)▶️Bleeding
  • Risk factors▶️placental injury (placental abruption), sepsis, postpartum hemorrhage, acute fatty liver of pregnancy, amniotic fluid embolism
How well did you know this?
1
Not at all
2
3
4
5
Perfectly
138
Q

Triad on ECG of Wolff-Parkinson-White syndrome

A

Delta wave
Short PR interval
Widened QRS

How well did you know this?
1
Not at all
2
3
4
5
Perfectly
139
Q

Locus of amyloid precursor protein (APP), in what cases it can increase the risk of alzheimer disease?

A
  • Long arm of chromosome 21
  • Processing by gamma and beta secretase▶️beta-amyloid⏭not enough cleared, insoluble fibrils▶️extracellular amyloid plaque▶️neurotoxic

*Also ⬆️levels of APP ⬆️risk of alzheimer dementia⏩trisomy 21

How well did you know this?
1
Not at all
2
3
4
5
Perfectly
140
Q

What genes participate and what are their functions in adenoma to carcinoma sequence of colon cancer?

A
  • APC▶️tumor suppressor gene▶️⬆️B-catenin▶️uncontrolled cell proliferation▶️progression from normal mucosa to small adenomatous polyp (adenoma)
  • KRAS▶️protooncogene▶️unregulated cell growth▶️increase size of adenoma
  • TP53▶️tumor suppressor gene▶️apoptosis of cells with damaged DNA (normally), mutation▶️allow cells with bad DNA enter to cell cycle▶️last “hit”▶️carcinoma (malignant transformation)
How well did you know this?
1
Not at all
2
3
4
5
Perfectly
141
Q

What is made of the fibrous cap of atheroma plaque and what cell is responsible of its formation?

A
  • Collagen, elastin, proteoglycans

- Vascular smooth muscle cells

How well did you know this?
1
Not at all
2
3
4
5
Perfectly
142
Q

What type of antibody is the Rheumatoid factor and against does it react?

A
  • IgM
  • Fc portion of human IgG▶️Formation immunocomplexes (IgM-IgG)▶️deposition at sinovium and cartilage▶️complement activation▶️chronic inflammation
How well did you know this?
1
Not at all
2
3
4
5
Perfectly
143
Q

What tumor serum marker is elevated in malignant testicular tumor (mainly in nonseminomatous germ cell tumors) and how can you suspect it and why?

A
  • hCG
  • Alpha subunits of TSH, LH, FSH and hCG are identical, and Beta subunit is very similar in TSH and hCG▶️⬆️⬆️hCG▶️⬆️T3, T4▶️paraneoplasic hyperthyroidism (also seen in gestational trophoblastic disease)

*Alpha-phetoprotein also ⬆️ but doesn’t cause hyperthyroidism

How well did you know this?
1
Not at all
2
3
4
5
Perfectly
144
Q

Cystic degeneration of putamen is characteristic of what disease?

A

Some progressive neurologic disease, but most likely▶️Wilson’s disease

How well did you know this?
1
Not at all
2
3
4
5
Perfectly
145
Q

Translocation associated to Burkitt lymphoma and their consequences in the cell.

A
  • t(8;14)▶️c-myc oncogene on heavy Ig chain
  • t(8;22)▶️c-myc oncogene on Ig lambda light chain
  • t(8;2)▶️c-myc oncogene on Ig kappa light chain
  • All result in overexpression of the oncogene
How well did you know this?
1
Not at all
2
3
4
5
Perfectly
146
Q

Key finding and description of stages of acute tubular necrosis

A
  • Granular (“muddy blrown”) casts
    1. Inciting event→36 hours, slight ↓ in urine output
  1. Maintenance phase (1-3 wks): follow 24-36 hrs, ↓urine output (oliguric), ↑BUN, Cr (BUN:Cr <15), high anion gap metabolic acidosis, ↑extracellular fluid volume, ↓Na, ↓Ca, ↑PO4, ↑Mg, ↑K, ↓urinary osmolarity (<350), ↑urinary Na (>30), ↑FeNa>1
  2. Recovery phase: follow 1-2 wks. Impair tubular reabsorption and concentrating ability→polyuric (>3L/day), hypotonic urine→↓Mg, ↓PO4, ↓Ca (deposition and ↓Vit D), ↓K (most serious complication of this phase), ↓BUN,Cr.
How well did you know this?
1
Not at all
2
3
4
5
Perfectly
147
Q

Common causes of acute tubular necrosis

A
  • Ischemic→↓renal blood flow (hypotension, shock, sepsis, hemorrhage, HF)►PCT, thick ascending limb→↑susceptible to injury
  • Nephrotoxic→Drugs (aminoglycosides, cisplatin, sulfonamides, methycilin, polymyxin), radiocontrast agents, heavy metals (lead, mercury, gold), organic solvents (carbon tetrachloride, chloroform, methyl alcohol), crush injury (myoglobinuria), etc►PCT susceptible to injury
How well did you know this?
1
Not at all
2
3
4
5
Perfectly
148
Q

Where is the origin of the stanford type B and A aortic dissection?

A
  • Type A→sinotubular junction►involve any part of ascending aorta
  • Type B→close to the origin of the left subclavian artery►involve the descending aorta (no ascending)
  • Aortic arch involvement→distal propagation of type A, proximal propagation of type B.
  • Thoracoabdominal aorta involvement→both type distal propagation
How well did you know this?
1
Not at all
2
3
4
5
Perfectly
149
Q

Clinical manifestation of Wiskott-Aldrich syndrome? Cause and mechanism of them.

A
  • X-linked disorder→combined B and T-lymphocyte dirsorder
  • Eczema
  • Recurrent infections→pyogenic infections→inability of hummoral response against organisms with polysaccharide capsule. T cell defect→oportunistic infections (herpes or pneumocystis)►after 6 to 12 mo of birth
  • Thrombocytopenia→petechiae, purpura, hematemesis, epistaxis
How well did you know this?
1
Not at all
2
3
4
5
Perfectly
150
Q

What type of acid-base disturbance can you find in septic shock?

A
  • High anion-gap metabolic acidosis, secondary to:
  • Lactic acidosis►End organ hypoperfusion→↓oxidative phosphorylation→↑NADH→shunt Pyr to Lac(↑)→continue glycolysis

*Hepatic hypoperfusion→↑Lactate►primary site of lactate clearance

How well did you know this?
1
Not at all
2
3
4
5
Perfectly
151
Q

Clinical clues to suspect acute interstitial nephritis, and most common causes.

A
  • Acute renal failure (azotemia), ↑IgE serum and eosinophilia, eosinophiluria, fever, maculopapular rash
  • 1-3 wks after initiation of: B-lactam antibiotic, diuretics, sulfonamides, rifampin, NSAIDS, proton pump inhibitors
How well did you know this?
1
Not at all
2
3
4
5
Perfectly
152
Q

What type of urinary incontinence is due multiple sclerosis and why?

A
  • Urge incontinence→detrussor overactivity
  • Regions in pons and cerebral cortex partially inhibit micturition reflex (also regulate contraction/relaxation of external urethral sphincter)→involved area in MS (above sacral region)→loss of higher center control→detrussor hyperreflexia´

*As disease progress bladder become atonic and dilated→overflow incontinence

How well did you know this?
1
Not at all
2
3
4
5
Perfectly
153
Q

What suggest a C1 that when is not attached to Ag-Ab complex is excessively cleaving C2 and C4 (inappropriate activation of the complement cascade)? why?

A
  • C1 inhibitor (C1INH) deficiency→normally blocks kallikrein-induced conversion of kininogen to bradykinin►potent vasodilator→↑vascular permeability
  • Facial swelling (without urticaria), laryngeal edema, GI symptoms (nausea, vomiting, colicky)
How well did you know this?
1
Not at all
2
3
4
5
Perfectly
154
Q

What is the manifestation of a enteropeptidase deficiency?

A

Fat and protein malabsorption→diarrhea, failure to thrive, edema (hypoproteinemia)

*Fail to produce trypsin→break polypeptides and activates enzymes for protein and lipid digestion (pancreatic enzymes)

How well did you know this?
1
Not at all
2
3
4
5
Perfectly
155
Q

What type of cells become hypertrophic and make hyperplasia in chronic renal hypoperfusion (stenosis)? Why?

A
  • Juxtaglomerular cells of the afferent arteriole (main location) [modified smooth muscle cells]
  • Hypoperfusion▶️➕RAAS▶️JG cell synthesize renin▶️chronic ➕▶️hypertrophy and hyperplasia
How well did you know this?
1
Not at all
2
3
4
5
Perfectly
156
Q

Mechanism by which rickets result

A

⬇️⬇️Vitamin D levels (⬇️conversion to active form or ⬇️intake)▶️⬇️Ca and PO4▶️unmineralized bone matrix and epiphyseal (growth plate) cartilage

How well did you know this?
1
Not at all
2
3
4
5
Perfectly
157
Q

In a severe hypotensive patient which zones of the colon would be mainly affected and why?

A

Nonocclusive ischemia at margins of anastomotic distributions▶️”watershed” areas▶️splenic flexure (border between SMA and IMA), rectosigmoid junction (border between sigmoid and superior rectal artery) [Left colon]

*Ischemia and necrosis of intestinal wall▶️colonoscopy⏩pale mucosa and petechial hemorrhage

How well did you know this?
1
Not at all
2
3
4
5
Perfectly
158
Q

What mean a gastric erosion?

*Mainly causes of gastric ulcers

A

Lesion that extend into (not completely through) the muscularis mucosa▶️acute erosive gastropathy⏩alcohol, tobaco, NSAIDs, surgical stress, head trauma (cushing), burns (curling)

*Completely through muscularis mucosa▶️ulcer (extend into the submucosal layers and muscularis propia)⏩1.Helicobacter pylori, 2.NSAIDs

How well did you know this?
1
Not at all
2
3
4
5
Perfectly
159
Q

Clinical manifestations of acute erosive gastropathy

A
  • Asymptomatic
  • Life threatening upper GI bleeding (melena, etc)
  • Nausea, vomiting, epigastric pain (most common)
How well did you know this?
1
Not at all
2
3
4
5
Perfectly
160
Q

Single most important risk factors for aortic dissection and aneurysm

A
  • Aortic dissection▶️Hypertension (intimal tears)

- Aortic aneurysm▶️Atherosclerosis (abdominal>thoracic)

How well did you know this?
1
Not at all
2
3
4
5
Perfectly
161
Q

Histologic findings of celiac disease

A

Villous atrophy, crypt hyperplasia, intraepithelial lymphocyte infiltration

How well did you know this?
1
Not at all
2
3
4
5
Perfectly
162
Q

What changes suggest an irreversible neuronal damage, and what is the response to repair it?

A
  • Neuronal shrinkage, intense cytoplasmic eosinophilia

- Gliosis→glial hyperplasia→astrocytic processes form closely connected meshwork→gliotic scar

How well did you know this?
1
Not at all
2
3
4
5
Perfectly
163
Q

Histolopathologic changes of acute tubular necrosis.

A
  • Maintenance phase
  • Proximal tubular cell balooning, vacuolar degeneration
  • Patchy loss of tubular epithelial cells
  • Denudation of basement membrane
  • Interstitial edema
  • Recovery phase
  • Re-epithelization of tubules (gradually)
How well did you know this?
1
Not at all
2
3
4
5
Perfectly
164
Q

What suggest calcium oxalate crystals in tubular lumen and/or urinanalysis in a patient with acute renal failure?

A
  • Ethylene glycol poisoning►metabolized by GI tract→glycolic acid (toxic to renal tubules) and oxalic acid (precipitation as calcium oxalate)
  • Acute tubular necrosis→high anion gap metabolic acidosis, osmolar gap
How well did you know this?
1
Not at all
2
3
4
5
Perfectly
165
Q

What type of alteration in bladder would you expect to find in multiple sclerosis? Why does it occur and what do you expect find in urodynamic study?

A

Few weeks after acute spinal cord lesion (MS attack)▶️Spastic bladder (bladder hypertonia in urodynamic study▶️little or no residual urine after emptying▶️poor distensibility)▶️⬆️urinary frequency and urge incontinence

*Upper motoneuron lesion▶️loss descending inhibitory control on bladder▶️doesn’t relax/distend

How well did you know this?
1
Not at all
2
3
4
5
Perfectly
166
Q

Cause of flaccid bladder. Common finding in urodynamic study.

A
  • Lower motor neuron lesion (ex, cauda equina syndrome)▶️urinary incontinence at the end of theh day
  • Large residual volume after emptying
How well did you know this?
1
Not at all
2
3
4
5
Perfectly
167
Q

What is the consequence of the change of Hb in sickle cell disease (HbS)?

A

HbS▶️glu (negative charge)⏩val (neutral charge) susbtitution (missense mutation)▶️6th aa in B-globin chain▶️hydrophobic interaction among Hb▶️HbS polymerization and RBC sickling

  • Promote by low oxygen levels, ⬆️acidity, dehydration
  • ↓Negative charge→move more slowly toward anode in electrophoresis
How well did you know this?
1
Not at all
2
3
4
5
Perfectly
168
Q

Consequence of the change in Hb on HbC disease.

A

HbC▶️glu (negative charged)⏩lys (positive charged) (missense mutation)▶️less severe symptoms than sickle cell disease▶️hydrophobic interactions between Hb doesn’t occur

*↓↓Negative charge→move even more slowly toward anode in electrophoresis

How well did you know this?
1
Not at all
2
3
4
5
Perfectly
169
Q

Mechanism of Zenker (false) diverticulum

A

Cricopharyngeal motor dysfunction▶️spasms, ⬇️relaxation of cricopharyngeal muscles during swallowing

How well did you know this?
1
Not at all
2
3
4
5
Perfectly
170
Q

Complications of subarachnoid hemorrhage.

*Cause of it.

A
  • Vasospasm (most frequent)▶️impaired brain autoregulation (prevent - Nimodipine)
  • Rebleeding (most dangerous, no most common▶️30%)
  • Hydrocephalus▶️irritation and dysfunction of arachnoid villi

*Rupture of berry aneurysm

How well did you know this?
1
Not at all
2
3
4
5
Perfectly
171
Q

Important histologic features of Schwannoma.

A
  • Antoni A pattern▶️⬆️cellularity regions⏩spindle cells, oval nuclei in palisading; interspersed;
  • Antoni B pattern▶️⬇️cellularity myxoid regions
  • Verocay bodies▶️nuclear free zones interspersed in Antoni A
  • S-100 (+)▶️neural crest cell origin (as melanoma)

*Most common site cerebellopontine angle at CN VIII. Peripheral nerves, nerve roots, CN (except II CN - covered by oligodendrocytes)

How well did you know this?
1
Not at all
2
3
4
5
Perfectly
172
Q

With which conditions is associated the xanthelasmas?

A

Primary biliary cirrhosis

How well did you know this?
1
Not at all
2
3
4
5
Perfectly
173
Q

Where do you find the amatoxins? How can cause alterations and what can cause? How do you diagnose?

A
  • Poisonous mushrooms (Amanita phalloides [death cap])▶️🚫RNA polymerase II▶️🚫mRNA
  • 6-24 hrs after ingestion▶️abdominal pain, vomiting, severe cholera-like diarrhea (blood and mucus), acute hepatic and renal failure
  • Urine alpha-amanitin
How well did you know this?
1
Not at all
2
3
4
5
Perfectly
174
Q

What is the acute/subacute clinical presentation of acute rheumatic fever and its chronic feature? Of the acute features which is the major morbidity?

A
  • Migratory arthritis, sydenham chorea, pancarditis (major morbidity▶️severe myocarditis▶️cardiac dilation▶️HF▶️death)
  • Chronic: mitral stenosis/regurgitation

*In acute setting inflammation of mitral valve▶️mitral regurgitation▶️new holosystolic murmur

How well did you know this?
1
Not at all
2
3
4
5
Perfectly
175
Q

Serum levels in primary osteoporosis of PO4, Ca, PTH.

A

Normal levels

How well did you know this?
1
Not at all
2
3
4
5
Perfectly
176
Q

What is the most common manifestation of primary osteoporosis and why?

A

Early postmenopausal osteoporosis involved cancellous bone▶️integrity of vertebral column⏩vertebral fractures

How well did you know this?
1
Not at all
2
3
4
5
Perfectly
177
Q

Distinctive histopathologic finding of hepatitis B infection

A

“Ground-glass” hepatocytes▶️Finely granular, diffusely homogenous, pale eosinophilic cytoplasm▶️accumulation of Hepatitis B surface antigen

How well did you know this?
1
Not at all
2
3
4
5
Perfectly
178
Q

What vitamins and compound deficiencies can you find after gastric pass procedures? Ex, Bilroth II (gastrojejunostomy)

A

Iron, vitamin B12, folate, vitamin D and calcium▶️predominantly absorbed at duodenum and proximal jejunum (leaved as blind loop in Bilroth II)

*As gastric Antrum is removed▶️⬇️gastrin▶️⬇️acid▶️⬇️iron absorption

How well did you know this?
1
Not at all
2
3
4
5
Perfectly
179
Q

Mutation associated with MEN2 syndromes. Why may you find episodic headache as one of symptoms in MEN2B?

A
  • Germline mutations of the RET protooncogene
  • Pheochromocytoma associated▶️episodic catecholamines secretion (also paroxysmal hypertension, flushing, diaphoresis)

*Medullary thyroid cancer (⬆️calcitonin), mucosal neuromas, marfanoid habitus.

How well did you know this?
1
Not at all
2
3
4
5
Perfectly
180
Q

Important features of vesicoureteral reflux (VUR) and why does it occurs?

A
  • Recurrent pyelonephritis▶️retrograde urine flow from bladder▶️renal scarring in upper and lower poles, dilated calyces, cortical atrophy bilaterally
  • Secondary hypertension
How well did you know this?
1
Not at all
2
3
4
5
Perfectly
181
Q

Presentation and cause of posterior urethral valve

A
  • Hydronephrosism, calyceal dilatation▶️obstruction of urine flow in the urethra
  • Malformation of wolffian duct▶️only occur in males
How well did you know this?
1
Not at all
2
3
4
5
Perfectly
182
Q

Which regions of the lymph nodes are absent in DiGeorge syndrome and agammaglobulinemia and why?

A
  • DiGeorge syndrome▶️⬇️T-cells▶️paracortex region

- Agammaglobulinemia▶️⬇️B-cells▶️germinal centers and primary lymphoid follicles

How well did you know this?
1
Not at all
2
3
4
5
Perfectly
183
Q

What could decrease the development of clinical peripheral edema in patient with COPD secondary cor-pulmonale?

A

Cor-pulmonale▶️right side heart failure▶️⬆️central venous pressure (CVP)▶️⬆️plasma hydrostatic pressure▶️transudation to interstitial tissue▶️⬆️interstitial hydrostatic pressure▶️⬆️lymphatic drainage

*If ⬆️⬆️CVP▶️interstitial hydorstatic pressure overwhelm lymphatic drainage capacity▶️edema

How well did you know this?
1
Not at all
2
3
4
5
Perfectly
184
Q

Clinical presentation of Pancoast tumor

A
  • Ipsilateral upper limb weakness, pain, ⬇️reflex, paresthesias▶️involvement of brachial plexus
  • Horner syndrome▶️cervical sympathetic ganglia involvement
185
Q

Factor associated in 90% of thrombus formation in nonvalvular atrial fibrillation

A

Left atrial appendage

186
Q

Why is giant cell arteritis considered a medical emergency? Why should you avoid with prompt treatment?

A

Irreversible blindness or ocular complications:

  • Ischemic optic neuropathy
  • Amaurosis fugax
  • Central or branch retinal artery occlusion
  • Cerebral infarction▶️central visual fields defects

*Not wait Bx to initiate high dose glucocorticoid therapy

187
Q

Why can you find osteomalacya in celiac disease? How do you expect find the PTH, Ca and PO4?

A
  • ⬇️ Absorption of nutrients and vitamins, as Vitamin D
  • ⬆️PTH, ⬇️Ca (⬇️intestinal absorption), ⬇️PO4 (⬇️interstinal absorption, ⬆️renal excretion)

*⬆️PTH is not enough to ⬆️Ca▶️specially when bone stores become depleted

188
Q

Histopathological finding of mesothelioma. Clinical presentation and risk factor associated.

A
  • Tumor cells with numerous, long, slender microvilli
  • Abundant tonofilaments→Pancytokeratin, calretinin (immunohistochemical marker)
  • Psamomma bodies
  • Pleural thickening (radiographic study), hemorrhagic pleural effusion
  • Asbestos exposure
189
Q

What type of DNA repair is altered in Xeroderma pigmentosum?

A

Nucleotide excision repair of DNA damaged by UV light

*⬆️Risk of skin malignancy▶️melanoma, squamous and basal cell carcinoma (early age 5-6)

190
Q

Effect of nitrites on Hb

A
  • Hem of Hb: Fe2+(reduced)▶️Fe3+(oxidized)[Methemoglobin]▶️unable to bind O2▶️⬇️O2 content and O2 carry capacity
  • Dusky skin coloration (similar cyanosis)
  • Blood partial pressure of O2▶️normal▶️measure O2 dissolved in plasma (unrelated to Hb function)
  • Also ⬆️O2 affinity of reduced Fe▶️left shift curve▶️⬇️delivery O2 to tissues.
  • Binds tightly to cyanide (important for cyanide poisoning)
191
Q

Special stain for inclusions seen in blood smear of glucose-6-P dehydrogenase deficiency. Inheritance pattern.

A
  • Supravital stain▶️dark, intracellular inclusions▶️Heinz bodies
  • X-linked recessive
192
Q

Key differentiation between TBC and sarcoidosis

A
  • Bx Sarcoidosis▶️non-caseating granulomas with epithelioid histiocytes, multinucleated giant cell of Langhans, Asteroid bodies, Schaumann bodies
  • Bx TBC▶️caseating granulomas and acid fast bacilli

*Sarcoidosis▶️bilaterally hilar adenopathy, pulmonary compliants and constitutional symptoms

193
Q

Most common cause of inherited trombophilia. Clinical presentation.

A
  • Factor V Leiden▶️glutamine for arginine near activated protein C cleavage site▶️⬇️susceptibility to be cleavage▶️hypercoagulable state
  • 1-9% of worlwide caucasian population are heterozygous▶️5-10 times risk of thrombosis, homozygous▶️50-100% risk
  • Deep venous thrombosis (50% untreated make pulmonary embolism), cerebral vein thrombosis, recurrent pregnancy loss
194
Q

Cause of familial pulmonary arterial hypertension (PAH). Histopathological findings.

*Classic presentation.

A
  • Mutation on pro-apoptotic BMPR2 gene→↑endothelial and smooth muscle proliferation►vascular remodeling, ↑vascular pulmonary resistance, progressive pulmonary hypertension
  • Medial hypertrophy, intimal hyperplasia, intimal fibrosis (onion skin), formation of capillary tufts (plexiform lesion)

*Dyspnea and exercise intolerance in women aged 20-40

195
Q

Which vitamins do you expect to find elevated after gastric by pass surgery? and why? Which nutrients may be deficient?

A
  • Small intestinal bacterial overgrowth (SIBO)→↑production of Folate and Vitamin K
  • Others fat-soluble vitamins (A, D, E), Iron, Zinc, Vit B12 (↓absorption free and intrinsic factor bound)
  • The close-ended gastroduodenal limb have a large bacterial proliferation
196
Q

Complication of long standing carcinoid syndrome

A

Right sided valvular heart disease

197
Q

What uterine fibroid can cause constipation?

A

Posterior subserosal fibroid leiomyoma→irregular uterine enlargement (pressure on adjacent organs)→bulk-related symptoms►pelvic pressure, colon→constipation

198
Q

What is the Bloom syndrome and its cause and clinical presentation?

A
  • BLM gene mutation→helicase (unwind dsDNA during replication) dysfunction→chormosomal instability and breakage
  • Mental retardation, facial abnormalities (microcephaly), photosensitivity rash, immunodeficiency (recurrent infections)
199
Q

What is myocardial hibernation?

A
  • Chronic myocardial ischemia→↓myocardial metabolism and function→prevents myocardial necrosis►↓contractility and left ventricular systolic dysfunction
  • Partially or completely reversible by coronary revascularization
200
Q

Most important nonmodifiable risk factor for osteoporotic fractures between females

A

Ethnicity→bone mass african-american>white, hispanic or asian→↓risk of fractures

201
Q

How do you identify a primary (psychogenic) polydipsia? How do you differentiate from diabetes insipidus?

A
  • Hyponatremia, low initial urine osmolality
  • Water deprivation test:
    🔵Psychogenic polydipsia▶️⬆️initial urine osmolality, no significant ⬆️urine osmlolality after ADH administration (maximal endogenous ADH effect)
    🔴DI▶️NO change in urine osmolality, ADH administration▶️⬆️⬆️urine Osm in central, ⬆️urine Osm in nephrogenic. [⬆️Na serum (nephrogenic and central)]

*Psychosocial disorder ⬆️intake of free water

202
Q

What is hydrocephalus ex-vacuo and in what cases may you see it?

A
  • Hydrocephalus by central neuronal volume loss
  • Neurodegenerative diseases (ex, AIDS dementia, Alzheimer, Huntington, Pick diseases)▶️cortical atrophy▶️ventricles expand with normal CSF pressure

*Triad is NOT seen

203
Q

Causes and clinical presentation of familial chylomicronemia syndrome (hyperlipoproteinemia type I).

A
  • Lipoprotein lipase (LPS) deficiency or defect Apo C-II
  • Recurrent acute pancreatitis, skin eruptive xanthomas, lipemia retinalis (milky appereance retinal vasculature), hepatosplenomegaly, ⬆️triglycerides (⬆️chylomicrons)
204
Q

Children with abdominal mass and opsoclonus-myoclonus syndrome is associated with what condition? Describe that syndrome.

A
  • Neuroblastoma (from neuroblast of adrenal medulla)→Neural crest cells
  • Nonrythmic conjugate eye movements in some directions, hypotonia, myoclonus (is a Paraneoplasic syndrome)

*Most common extracranial (and of the adrenal medulla) childhood cancer

205
Q

Histologic finding of subacute granulomatous (de Quervain) thyroiditis

A
  • Early neutrophilic infiltrate (microabscess)

- Mixed inflammatory cells, Macrophage infiltrate and multinucleated giant cells

206
Q

Which finding may suggest an underlying cause of hyperthyroidism?

A

Pretibial myxedema and exophtalmos

*Autoimmune response against TSH receptor in pretibial connective tissue and periorbital▶️⬆️T lymphocyte infiltration▶️CK’s▶️⬆️GAG’s and fibroblast proliferation▶️mucinous edema and tissue expansion

207
Q

Most common cause of nephrosis in children

A

Minimal change disease

208
Q

Patient with abdominal pain nausea, abdominal CT with intestinal wall thickening and no enhancement with contrast, metabolic acidosis with high lactate, atrial fibrillation. Diagnose and management.

A
  • Acute ischemic colitis▶️embolic due Afib
  • Ischemic bowel▶️anaerobic metabolism▶️⬆️lactate▶️anion gap metabolic acidosis▶️➕renal ammoniagenesis▶️Glutamine (glutamate)▶️⬆️ammonium [responsible of acid excretion in acidosis, most chronic acidosis] (tubular fluid - excreted), HCO3 (reabsorbed - buffer)
209
Q

Histologic finding of urticaria

A

Dermal edema▶️⬆️permeability of microvasculature▶️edema of superficial dermis

*Involvement of deep dermis▶️angioedema

210
Q

Finding at urine sediment pathognomonic of pyelonephritis

A

WBC casts▶️only formed at tubules▶️precipitate with Tamm-Horsfall protein (secreted by tubular epithelial cells)

*also seen in acute interstitial nephritis

211
Q

Etiology and histologic finding of molluscus contagiosum.

A
  • Poxvirus (MC virus)▶️umbilicated, firm, flesh-colored lesions in skin and mucosal membranes
  • Epidermal hyperplasia, large intracytoplasmic eosinophilic inclusions (molluscus bodies - virus particles)
212
Q

Histologic change of lung adenocarcinoma in situ, behavior and location. Frequency.

A
  • Well-differentiated dysplastic cuboidal cells lining the alveolar tissue with or without intracellular mucin
  • Preinvasive lesion▶️growth along intact alveolar septa, no vascular or estromal invasion
  • Located at periphery of lung
  • Suptype of adenocarcinoma (most common lung cancer 40-50%, except for metastases) former bronchioloalveolar carcinoma (itself <10%)
213
Q

Which types of hematologic Neoplasias do you suspect if they are due to a mutation in a cytoplasmic non-receptor tyrosine kinase that ⬆️its activity and then also ⬆️STAT activation? What is the exception of these types of neoplasias, why?

A

Myeloproliferative syndromes

  • Policytemia vera, essential thrombocytosis, myelofibrosis▶️JAK mutation▶️➕STAT▶️⬆️transcription
  • Chronic myelogenous leukemia▶️t(9;22) BCR-ABL▶️Philadelphia chromosome
214
Q

In infectious mononucleosis which type of cell is infected primarily and which do you find in peripheral blood smear as atypical cell?

A
  • B-cell▶️EBV infects by CD21

- Activated CD8 cytotoxic T-cell▶️reaction to destroy infected cell▶️atypical reactive in blood smear

215
Q

What disease cause a point gene mutation of neurophysin?

A

Carrier proteins for oxytocin and vasopressin▶️Diabetes insipidus

*Mutation in neurophysin II▶️most cases of hereditary hypothalamic diabetes insipidus▶️⬇️release of ADH into systemic circulation

216
Q

Etiology of encephalitis that finding is most commonly unilateral but also bilateral hemorrhagic necrosis of temporal lobe. Which symptoms do you expect to find with that involvement? Diagnosis.

A
  • HSV-1
  • Personality change (hypersexuality, aggression), aphasia▶️temporal lobe involvement
  • PCR in CSF
  • Primary oropharyngeal infection or reactivation of latent virus
  • Classic features of encephalitis▶️fever, seizures, change mental status, headache
217
Q

Extraskeletal involvement of ankylosing spondylitis

A
  • Respiratory▶️limit chest wall expansion (costovertebral and costosternal junctions enthesopathies, thoracic spine involvement)▶️hypoventilation (monitor disease)
  • Cardiovascular▶️ascending aortitis (aortic insufficiency)
  • Eye▶️anterior uveitis
218
Q

What is the cause of Meniere disease? Most important features.

A
  • Increase volume and pressure of endolymph

- Recurrent Vertigo, Tinnitus, Unilateral sensorineural hearing loss, ear fullness/pain

219
Q

What is the factor V Leiden mutation and its consequence?

A
  • Factor Va resistant to the cleavage (inactivation) by activated Protein C
  • 20% of atypical venous thrombosis
220
Q

Most common causes of inherited hypercoagulability.

A
  • Factor V Leiden mutation

- Mutation in Prothrombin gene

221
Q

Indication and mechanism of action of Ramelteon.

A
  • Melatonin agonist▶️binds with ⬆️affinity to melatonin receptors in suprachiasmatic nucleus
  • Safety and efficacy in older adults for insomnia management. No dosis adjustment needed
222
Q

Symptoms and cause of Vitamin E deficiency. Why can you suspect it in cystic fibrosis?

A

Vitamin E▶️protects against fatty acid oxidation
- Deficiency▶️⬆️susceptibility of neuronal and RBC membranes to oxidative stress
🔴Neuromuscular disease▶️ataxia, hyporeflexia, impaired proprioception and vibratory sensation
🔵Hemolytic anemia
- Exocrine pancreatic deficiency (can occur in CF)▶️fat malabsorption (lipid-solubles vitamins)

223
Q

Deficient neurotransmitter in Huntington disease and gross changes in brain.

A
  • Loss of GABA-containing neurons▶️⬇️GABA

- Bilateral atrophy of caudate nucleus and putamen (both form striatum), dilation of frontal horns of lateral ventricles

224
Q

Histolopathological finding that show the underlying cause of abdominal aortic aneurysm formation.

A

Chronic transmural inflammation

*Oxidative stress and vascular smooth muscle apoptosis

225
Q

What lipid lowering agent should be avoided in a patient with gallbladder stones?

A

Gemfibrozil (fibrates)▶️🚫cholesterol 7-alpha hydroxylase▶️⬇️bile acid synthesis▶️⬇️cholesterol solubility▶️⬆️cholesterol secretion and microcrystals precipitation▶️gallstone formation

226
Q

Most vulnerables areas or brain to ischemia. Firsts with irreversible damage.

A
  • Pyramidal neurons of Hippocampus (1st) and neocortex
  • Purkinje neurons of cerebellum
  • “Watershed infarction”▶️gray matter between perfusion zones of major cerebral arteries

*Global cerebral ischemia [hypoxic-ischemic encephalopathy] (ex, cardiac arrest)▶️if Ischemia lasting >5 minutes▶️irreversible damage

227
Q

Mechanism of poison ivy, sumac or oak dermatitis. What cell mediates it?

A
  • Produce urushiol→allergen→bind to protein (hapten)►type IV (delayed type) hypersensitivity reaction→allergic contact dermatitis
  • CD8 T cells→primary effector►destroy keratinocytes expressing haptenated proteins
228
Q

Which findings can you see in a patient with chronic alcoholic pancreatitis and why?

A
  • Alcohol-induced secretion of protein-rich fluid→ductal plugs in pancreatic ducts→calcify►epigastric calcifications on abdominal imaging (CT or x-ray)
  • Exocrine insufficiency→malabsorption→diarrhea/steatorrhea, weight loss
229
Q

Cell population that compound the primary central nervous system lymphoma, etiology or association in most cases? What other neoplasm you may differentiate?

A
  • PCNSL▶️B-lymphocytes▶️EBV in CSF in most cases. late complication of HIV infection
  • AIDS associated T-cell lymphoma▶️T-lymphocytes
230
Q

What pathway allow the spread of prostate (also pelvic) cancer to lumbosacral spine?

A

Prostatic venous plexus▶️vertebral venous plexus

*Hematogenously. After lymph nodes spread (lymphatic), liver, lung and skeletal is the 4th most common site of metastases.

231
Q

What you should rule out in a patient with an unexplained creatinine kinase elevation, chronic fatigue and mild weight gain? How is the clinical presentation?

A
  • Hypothyroidism▶️Hypothyroid myopathy▶️myalgia, proximal muscle weakness, cramping
  • Myoedema▶️slow reabsorption of calcium by the sarcoplasmic reticulum⏩focal mounding of muscle following percusion
232
Q

Toxic mechanism of arsenic poisoning. Classic finding to physical examination.

A
  • Binds to sulfhydryl groups🚫Pyruvate dehydrogenase▶️🚫celullar respiration
  • 🚫gluconeogenesis and glutathione metabolism
  • Garlic odor in breath or stool
233
Q

Important finding of arsenic poisoning that can lead in cardiac complication. Physical finding that suggest it. Treatment of the poisoning.

A
  • QTc prolongation▶️torsades de pointes
  • Garlic breath
  • Dimercaprol▶️sulfhydryl groups binds to arsenic and displaces it from enzymes of cellular respiration▶️⬆️nontoxic soluble chelates▶️⬆️urinary excretion of heavy metals
234
Q

What is the Ortner syndrome and its cause?

A

Mitral stenosis▶️left atrial dilatation▶️impinge of left recurrent laryngeal nerve▶️hoarseness

235
Q

Mechanism of clinical presentation of VIPoma.

A
  • Watery diarrhea, hypokalemia, achlorhydria (WDHA) syndrome
  • VIP▶️⬆️pancreatic HCO3 and clorhide secretion▶️bind to intestinal epithelial cells▶️➕adenylate cyclase▶️⬇️cAMP▶️Na, K, Cl and water secretion into bowel [secretory diarrhea]
236
Q

Complication of nephrotic syndrome that lead in hematuria, flank pain, membranous nephropathy and new onset left-side varicocele. How is the LDH?

A
  • Renal vein thrombosis▶️loss anticoagulant factors, especially antithrombin III
  • Renal infarction▶️⬆️LDH
237
Q

Potential procedure complications during resection and hydatid cysts from liver? Organism associated?

A
  • Spilling of cyst content▶️anaphylactic shock

- Echinococcus granulosum (unilocular cyst), multilocularis (multilocular cysts)

238
Q

Components of Reye syndrome and histological finding.

A
  • Hepatic dysfunction▶️⬆️AST, ALT, ammonia, bilirrubin, PT, PTT [vomiting, hepatomegaly]⏩Microvesicular steatosis without inflammation
  • Encephalopathy▶️secondary hepatic dysfunction▶️hyperammonemia in CNS▶️cerebral edema [confusion, coma]
239
Q

Deficiency in which enzymes cause xeroderma pigmentosum? Risk of which neoplasias? Clinical presentation.

A
  • Endonucleases▶️repair thymine dimers caused by UV light exposure⏩defect nucleotide excision repair
  • Squamous cell carcinoma
  • Children with photosensitivity, poikiloderma, hyperpigmentation in sun exposed areas
240
Q

What do spider angiomata, gynecomastia, testicular atrophy and decreased body hair have in common in liver cirrhosis?

A
  • Hyperestrinism→↑adrenal production of androstenedione→↑estrone (aromatization)→↑estradiol→↑sex hormone binding globulin→prefer bind testosterone►↓free testosterone/estrogen ratio
  • ↓Estrogen metabolism by the liver
241
Q

What are the Döhle bodies? In which situations can you see them?

A
  • Light blue (basophilic) peripheral granules in neutrophils→ribosomes bound with RER
  • Toxic systemic illness, burns or myelodysplasia
242
Q

What is the leukemoid reactions? Important laboratory to diagnose it.

A
  • Benign leukocytosis (>50.000/mm3) in response to underlying condition (severe infection, hemorrhage, solid tumor, acute hemolysis)
  • Serum leukocyte alkaline phosphatase normal or ↑ (↓ in chronic myelogenous leukemia)
243
Q

What is the Kussmaul sign? what disease does it suggest?

A
  • Abnormal rise in jugular venous pressure (JVP) during inspiration→volume restrictred ventricle unable to accommodate the respiratory ↑ in venous return
  • Constrictive pericarditis

*Normally JVP drops during inspiration

244
Q

Common structure of brain affected in contralateral arm, leg, lower face pure motor weakness, “clasp-knife” spasticity, hyperreflexia, ↑tone, positive babinski sign.

A

Internal capsule stroke

245
Q

What suggest to you a proggresively weakening of diaphragmatic contractions during maximal voluntary ventilation and intact phrenic nerve stimulation?

A
  • Neuromuscular junction pathology (myasthenia gravis)

- Abnormally rapid diaphragmatic muscle fatigue (restrictive lung or chest wall disease)

246
Q

What is the difference in the esophageal manometry between achalasia and scleroderma?

A
  • Achalasia▶️⬇️amplitude of peristalsis in mid esophagus, ⬆️tone and incomplete relaxation of lower esophageal esphincter
  • Scleroderma▶️⬇️peristalsis, ⬇️tone and incompetence of LES
247
Q

Laboratory findings of hemophilia A or B

A
  • Normal bleeding time, platelet count and PT (“PeT”▶️extrinsic pathway▶️II, V, VII, X)
  • ⬆️PTT▶️⬇️factor VIII or IX (“PiTT”▶️intrinsic pathway▶️XII, XI, IX, VIII, and common pathway▶️II, V, X)
248
Q

How are the levels of testosterone, LH, FSH and estradiol in klinefelter syndrome and why? Which clinical presentations can be explained by them?

A
  • Primary hypogonadism, azoospermia
  • Atrophied, hyalinized, seminiferous tubules→↓Inhibin►no negative feedback→↑LH, FSH→↑Estrogens (estradiol)→gynecomastia
  • Damaged Leydig cells→↓Testosterone→absent secondary male characteristics

*↑long bone length, learnin and socialization disabilities

249
Q

Histological finding of vitiligo, association with which type of diseases that make to suspect it.

A
  • Absence of melanocytes in the skin and complete absence of melanin pigment
  • Autoimmune disorders→type I DM, pernicious anemia, addison disease, autoimmune hepatitis, grave’s disease, autoimmune thyroiditis
250
Q

First and second trimester screening results for Down syndrome

A
  • First-trimester
    Ultrasound▶️nuchal translucency and hypoplastic nasal bone, ⬇️PAPP-A, ⬆️free β-hCG.
  • Second-trimester quadruple screen
    ⬇️α-fetoprotein, ⬆️β-hCG, ⬇️estriol, ⬆️inhibin A (Accurate dating is important)
251
Q

Histologically how can you differentiate primary sclerosing cholangitis (PSC) from primary biliary cirrhosis (PBC)? Another clinical clue of both that allow distinguishing?

A
  • PSC▶️diffuse inflammation, fibrosis of large extra and intra hepatic bile ducts with periductal concentric fibrosis (onion skin), small duct obstruction⏩Ulcerative colitis, ⬆️alkaline phosphatase.
  • PBC▶️lymphocyte infiltration, granulomatous destruction of intrahepatic interlobular bile ducts (“floride duct lesion)⏩fatigue, pruritus, cholestasis, xanthelasma
252
Q

Most common causes of toxic megacolon

A
  • Life-threatening complication of inflmmatory bowel disease (most ulcerative colitis than crohn disease)
  • Clostridium difficile infection and other forms of infectious colitis
253
Q

What suggest a mass from the brain that reports dilated capillaries with endothelial lining separated by a thin layer of fibrous connective tissue?

A

Cavernous hemangioma▶️recurrent hemorrhage⏩neurological deficits and seizures

254
Q

Synaptophysin is a marker of what type of cells?

A

Neuronal origin of CNS tumor▶️rare, <1% of primary.

*Transmembrane glyprotein of presynaptic neuronal vesicles, neuroendocrine and neuroectodermal cells

255
Q

Which lymphoma can present clinically with painless waxing and waining lymphadenopathy and clinical course?

A

Follicular lymphoma

  • Abdominal discomfort from abdominal mass
  • t(14;18) bcl-2 oncogene
256
Q

Histopathology of follicular lymphoma

A

Mixed of cleaved and noncleaved follicle center cells in a nodular pattern

257
Q

How do you explain hypercalcemia in sarcoidosis and others granulomatous disorders?

A

⬆️PTH independent conversion to 1,25 DiOH vit D by alpha-1 hydroxylase in activated macrophages in lung and lymph nodes

258
Q

Why may you find pancreatic islet amyloid deposition in diabetes type 2?

A

⬆️Amylin▶️islet amyloid polypeptide, co-secreted with insulin

*May play role in insulin secretion and beta cell apoptosis

259
Q

Etiology and clinical presentation of autosomal dominant polycystic kidney disease (ADPKD)

A
  • Mutation in PKD1 (85% of cases, chromosome 16) or PKD2 (15% of cases, chromosome 4)
  • Flank pain, hematuria, hypertension, urinary infection, progressive renal failure in ~ 50% of individuals
  • Associated with berry aneurysms, mitral valve prolapse, benign hepatic cysts

*Most common hereditary cause of renal failure in adults

260
Q

Common finding of diabetic nephropathy in light microscopic histopathology sample

A
  • Kimmelstiel-wilson nodules▶️nodular glomerulosclerosis
  • GBM thickening, ⬆️mensangial matrix deposition▶️progressive expansion of mensagium▶️nodules compress glomerular capillaries▶️⬇️GFR, nephrotic syndrome, HTN, renal failure
261
Q

What is the Reid index and for what is used?

A
  • Thickness of mucosal gland layer in the bronchial wall submucosal (major contributor of wall thickening) to thickness of wall between epithelium and cartilage
  • Sensitive measure of mucous gland enlargement in chronic bronchitis
  • > 0,4 or 40%▶️correlation with duration and severity of chronic bronchitis
262
Q

In what context and clinical presentation do you suspect a Fat embolism? Histopathology finding.

A
  • Severe long bone and/or pelvic fractures
  • Neurologic abnormalities (altered mentation, seizures), hypoxemia (dyspnea, tachypnea), petechial rash→classic triad
  • Fat microglobules in pulmonary microvessels (traumatic dislodged from bone marrow)
  • Common 24-72 hours after injury, but can occur until 2 weeks later
263
Q

Which are the 3 main causes of HIV-associated esophagitis and their pathologic findings?

A
  • Candida albicans (most common)▶️patches of adherent white/gray pseudomembranes
  • HSV▶️small vesicles▶️”punched out” ulcers
  • CMV▶️linear ulcerations
264
Q

Which pathological finding do you expect to find in CNS of PKU patient? Some clinical features.

A
  • Pallor of substantial nigra, locus ceruleus, vagas nucleus dorsalis (catecholaminergic)
  • Severe intellectual disability, musty body odor, seizures, hypopigmentation of hair, eyes, skin
265
Q

What overreactive enzyme can explain an increase risk to develop benz(o) pyrene induced lung cancer? Why?

A

Microsomal cytochrome P450 monooxygenase (hepatic microsomes and endoplasmic reticula of other tissues)▶️converts pro-carcinogens➡️carcinogens▶️DNA mutations

266
Q

In what case you may find a hypothyroidism with not elevated TSH level?

A
  • Secondary or tertiary (central) hypothyroidism▶️central is rare <5%
  • Findings indicating pituitary or hypothalamic dysfunction
267
Q

Structures affected, symptoms and pathogenesis of HIV associated dementia

A
  • Subcortical deep gray structures▶️subcortical dementia▶️attention/working memory problems, executive dysfunction, slow information processing
  • Inflammatory activation of macrophages and microglial cells▶️form groups (microglial nodules) around small areas of necrosis, may fuse▶️mutinucleated giant cells▶️⬆️Ck’s, HIV proteins▶️neuronal damage
268
Q

Which important acid-base and electrolytic disturbance may you find in cystic fibrosis and why? what drug adverse effect remember that alteration?

A
  • Contraction alkalosis and hypokalemia→ECF losses H2O/Na+ (are reabsorbed by epithelial cells, put inside cell)→concomitant renal K+/H+ wasting
  • Effect analogous to a loop diuretic
269
Q

Screening test for cystic fibrosis in a newborn

A

↑ serum Immunoreactive trypsinogen or trypsin levels

270
Q

What is the consequence to correct rapidly chronic hyponatremia? Clinical presentation.

A
  • Central pontine myelinolysis
  • Spastic quadriplegia→demyelination of the corticospinal tracts
  • Pseudobulbar palsy (nead and neck muscle waekness, dysphagia, dysarthria)→demyelination of the corticobulbar tracts of CN IX, X, XI

*Pseudobulbar because nulei of cranial nerves are intacta; bulbar palsy→injury of their nuclei

271
Q

What is the presentation of minimal change disease? Findings in renal biopsy.

A
  • Most common cause of nephrotic syndrome in children
  • Light Microscopy→normal glomeruli (lipid may be seen in PCT cells), IF ⊝
  • Electron Microscopy→effacement and fusion of podocyte foot processes
272
Q

Pathogenesis of minimal change disease

A
  • Inciting event→upper respiratory infection, immunization, insect bite
  • Systemic T cell dysfunction→glomerular permeability factor→CK damage podocytes→↓anionic properties of glomerular basement membrane
273
Q

Clinical presentation and complications of Hemochromatosis

A
  • Micronodular cirrhosis, diabetes mellitus, skin pigmentation (“bronze diabetes”)
  • ↑Risk hepatocellular carcinoma, congestive heart failure, testicular atrophy/hypogonadism
274
Q

What is the pathogenesis of Hemochromatosis?

A

Mutation of HFE protein at enterocytes and hepatocytes→associated with transferrin receptor►sensor falsely ↓ iron stores:

  • Enterocytes→↑divalent metal transporter 1 (DMT1)→↑iron absorption
  • Hepatocytes→↓Hepcidin→↑Ferroportin on enterocytes→↑iron secretion into circulation

*Result►Iron overload

275
Q

Histolopathologically which disease looks like primary biliary cirrhosis (cholangitis)?

A

Graft-versus-host disease▶when involve liver, ⬆alkaline phosphatase, lymphocytic infiltration and destruction of small intrahepatic bile ducts

276
Q

During wound healing, what is the consequence of execessive matrix metalloproteinase activity and myofibroblast accumulation?

A

Contracture▶deformities of the wound and surrounding tissues (palms, soles, anterior thorax, serious burn sites)

277
Q

Most common cause of bacteremia and osteomyelitis in sickle cell disease.

A
  • Bacteremia▶Encapsulated organism: 1. Streptococcus pneumoniae, 2. Haemophilus influenza
  • Osteomyelitis▶1. Salmonella, 2. Staphylococcus aureus, 3. E. coli
278
Q

Why does hypercalcemia in sarcoidosis occur?

A

PTH-independent production of 1,25 dihydroxyvitamin D in activated macrophages of lungs and lymph nodes (1-alpha hydroxylase)

279
Q

Pathogenesis of normal pressure hydrocephalus

A
  • Elderly, idiopathic (50%)►Expansion of ventricles distorts the fibers of the corona radiata
  • Secondary causes
    (a) Prior subarachnoid hemorrhage
    (b) Prior intracranial surgery
    (c) Prior trauma
280
Q

Clinical presentation of normal pressure hydrocephalus

A

Classic triad:

  • Gait difficulties▶ataxia→”magnetic gait”
  • Cognitive disturbances▶concentration, attention, lack interest (potential cause of reversible dementia with shunting)
  • Urinary incontinence▶late manifestation, urgency type, ⬇inhibitory control of bladder contractions by the frontal lobe
  • “Wet, wobbly, and wacky”
  • Also upper motor neuron signs▶spasticity, hyperreflexia
281
Q

Which hematologic neoplasia express CD19 and coexpress CD5 on the lymphocytes?

A

CD19 (B-cell marker), CD5 (T-cell marker)

Mature B-cell leukemia (chronic lymphocytic leukemia/small lymphocytic lymphoma, CLL/SLL)

282
Q

Dystrophic calcification is a hallmark of which pathologic process?

A
  • Cell injury and death in all types of necrosis→coagulative, fat, caseous, liquefactive
  • Occur in normal calcium levels
283
Q

What is the pathogenesis of dystrophic calcification in cardiac valves?

A
  • Chronic hemodynamic stress or atherosclerotic inflammation→endothelial and fibroblast death►release of cellular degradation products into valvular interstitium→calcification
  • Aortic sclerosis→benign at elderly; Calcific aortic stenosis→progressive valvular stiffening over time
284
Q

When and where occur the metastatic calcification?

A

Hypercalcemia→calcification in normal tissues and organs→more alfalike►kidneys, lungs, systemic arteries, gastric mucosa

285
Q

What suggest a patient with late-onset asthma, rhinosinusitis, eosinophilia? Frequent laboratory finding.

A
  • Churg-Strauss→small to medium vessel vasculitis

- p-ANCA→antibodies against neutrophil myeloperoxidase

286
Q

How do you explain a wrist drop sign in a patient with Churg-Strauss?

A

Vasculitis affecting epineural vessels→asymmetric multifocal neuropathy►Mononeuritis multiplex
- Wrist drop→radial nerve involvement

287
Q

What is pathognomonic in carcinoid heart disease? how is it caused?

A
  • Plaque-like deposits of fibrous tissue at endocardium of right heart→tricuspid regurgitation, pulmonic valvulopathy, right-sided heart failure
  • ↑5HT→(+) fibroblast growth and fibrogenesis
288
Q

Skeletal findings in primary hyperparathyroidism (PHP)

A

Osteitis fibrosa cystica→subperiosteal erosions in phalanges, granular “salt and pepper” skull, osteolytic cysts in the long bones (brown tumor bone cysts)

289
Q

Which clues do you have to differentiate chronic myelogenous leukemia or leukemoid reaction? what do they have in common?

A

In common→↑WBC count (>50.000), ↑precursors forms (bands, metamyelocytes, myelocytes)

  • CML→uncontrolled mature granulocyte production►↓leukocyte (neutrophil) alkaline phosphatase, “myelocytic bulge” (myelocytes>metamyelocytes (more mature forms), bashophilia, eosinophilia
  • Leukemoid reaction→↑ or normal leukocyte (neutrohil) alkaline phosphatase
290
Q

Thickened valve leaflets (typically mitral and/or aortic), with small multiple vegetations on both surfaces that are composed by sterile platelet thrombi intertwined with strands of fibrin, immune complexes and mononuclear cells

A

Verrucous (Libman-Sacks) endocarditis

*Less commonly associated with SLE

291
Q

Skin and mucosal telangiectasias with recurrent severe nosebleeds

A

Osler-Weber-Rendu syndrome (hereditary hemorrhagic telangiectasia)→lips, oronasopharynx, respiratory tract, urinary tract, GI tract; rare brain, liver and spleen

*Rupture telangiectasia→epistaxis, GI bleeding, hematuria

292
Q

Rare congenital neurocutaneous disorder that shows cutaneous facial angiomas and leptomeningeal angiomas

A

Sturge-Weber syndrome (encephalotrigeminal angiomatosis at V1 and V2)→mental retardation, seizures, hemiplegia, skull radiopacities (tram-track calcifications in radiographs)

293
Q

What disease show in addition of myotonia, cataracts, frontal balding and gonadal atrophy? Pattern of inheritance. Finding at biopsy.

A
  • Myotonic dystrophy
  • Second most common inherited muscle disorder (Duchenne muscle dystrophy 1st)►autosomal dominant, abnormal trinucleotide repeat expansion (CTG)
  • Atrophy of type 1 muscle fibers
294
Q

Characteristic features of polycythemia vera and pathogenesis

A
  • ↑RBC mass, ↑plasma volume, ↓EPO
  • ↑platelets, WBC, thrombotic events, peptic ulcer and pruritus, gouty arthritis. Ruddy face, splenomegaly
  • 95%→V617F mutation→JAK2 gene►replace Val with Phe→hematopoeitic cells more sensitive to growth factors (EPO, thrombopoietin)
295
Q

What disease is strogly associated to dermatitis herpetiformis? what is that? and How does it occur?

A
  • Celiac disease→IgA antibodies against gliadin→crossreact with epidermal transglutaminase
  • Dermatitis herpetiformis→erythematous pruritic papules, vesicles, bullae, bilaterally at extensor surfaces (elbow, knee), upper bakc and buttocks►microabscesses of fibrin and neutrophils at dermal papillae tips
296
Q

Histopathologic finding of celiac disease

A

↑Intraepithelial lymphocytes, loss of villus height (atrophy), crypt hyperplasia

297
Q

Which studies results may suggest you a lactose intolerance?

A
  • ↓Stool pH
  • ↑Stool osmolality
  • ↑breath hydrogen content

*Fermentation of undigested lactose by gut bacteria→↑short chain fatty acids (acetate, butyrate, propionate)►acidify stool; ↑H+ gas (breath content)

298
Q

Pathogenesis and clinical presentation of chronic mesenteric ischemia

A
  • Atherosclerosis of mesenteric or celiac arteries→Dx by ultrasound or angiography►high grade stenosis
  • Intestinal hypoperfusion specially 1 hour after meals (blood is more needed for absorption)►”intestinal angina”→postprandial epigastric pain, food aversion, weight loss

*History of generalized atherosclerosis

299
Q

What si the cause of fistulas in chron disease? Are they present in ulcerative colitis?

A
  • Transmural inflammation→lesions affect the entire thickness of the bowel wall
  • Not seen in ulcerative colitis
300
Q

Which are the following steps to stablish the cause of metabolic alkalosis and why?

A
  • Volume status
  • Urine chloride:
  • Low <10mEq/L→Vomiting, nasogastric suctioning/prior diuretic use►saline responsive
  • High >20mEq/L→current diuretic use►saline responsive; Mineralocorticoid excess state (primary hyperaldosteronism [Conn Sx], primary hypercortisolism [Cushing Sx], ectopic ACTH production)►saline unresponsive
301
Q

What is the cause of increase erythrocyte sedimentation rate (ESR) during inflammation?

A
  • ↑TNF-alpha, IL-1, IL-6→liver produce acute phase reactants→fibrinogen, ferritin, serum amyloid A, serum amyloid P, complement
  • ↑Fibrinogen→RBC form stacks→sediment at faster rate than individual RBC
302
Q

Pathologic finding of perpetuate malignant hypertension

A

Hyperplastic arteriolosclerosis→onion-like concentric thickening of arteriolar walls (laminated smooth muscle cells and reduplicated basement membranes)►renal arterioles and elsewhere

303
Q

Which encephalic structure is probably damage in hemibalismus and a possible cause?

A
  • Subthalamic nucleus

- Lacunar stroke→consequence of long-standing hypertension and DM

304
Q

Common extrahepatic disease manifestations in HCV-positive patients

A
  • Membranous glomerulopathy

- Mixed cryoglobulinemia

305
Q

What neoplasias are associated with EBV infection?

A
  • Hodgkin and non-Hodgkin lymphomas (Burkitt lymphoma)
  • Nasopharyngeal carcinoma
  • CNS lymphoma (with HIV positive)
306
Q

What is the deficient enzyme and clinical manifestations of porphyria cutanea tarda?

A
  • Uroporphyrinogen decarboxylase→inherited or acquired (alcohol, iron, halogenated hydrocarbons, smoking, HCV, HIV
  • Photosensitivity→vesicles and blister formation in sun exposed areas

*Most common disorder of porphyrin synthesis

307
Q

Enzyme deficiency in acute intermittent porphyria. Clinical manifestations.

A
  • Porphobilinogen deaminase
  • 5 P’s: Painful abdomen, port-wine urine, polyneuropathy, neuroPsychiatry disturbances (metabolite buildup), precipitated by drugs (cytochrome P450 inducers), alcohol, starvation

*NO photosensitivity

308
Q

What can cause a intrapituitary hemorrhage, and which is the preexisting condition? Treatment.

A
  • Pituitary apoplexy→severe headache, bitemporal hemianopsia, ophtalmoplegia; panhypopituitarism►↓ACTH→adrenocortical insuffiency→cardiovascular collapse (life-threatening hypotension)
  • Pituitary adenoma→mild chronic headache, ↓libido
  • Tx→emergency neurocx and glucocorticoid replacement
309
Q

Typical histological findings of alzheimer disease

A
  • Neuritic (senile) plaques→extracellular, medial temporal lobe, central amyloid beta (AB) core sorrounded by dystrophic neurites→toxic. [secondary to impair clearance or overproduction of APP]
  • Neurofibrillary tangles→intracellular, hyperphosphorylated tau protein (normally microtubule stabilization)
310
Q

Effect of hypovolemia in GFR, RPF, FF, and why?

A
  • ↓↓RPF→↓GFR, but ↑FF
  • Low GFR→↓NaCl at macula densa→(+)RAAS→Ag II→Efferent arteriolar constriction (also afferent arteriolar dilation)→restore GFR (but still ↓)→↑FF
311
Q

What enzyme can unstabilized a atherosclerotic plaque? what is the consequence?

A

Chronic inflammatory progression of atheroma→↑Metalloproteinases→breakdown extrecellular matrix proteins►thin-cap fibroatheromas

*Likelihood of plaque rupture most related to plaque stability than size or degree of luminal narrowing→mechanical strenght of fibrous cap (thin>unstable)

312
Q

Classic triad of proxysmal nocturnal hemoglobinuria. What is the renal involvement and why does it happen?

A
  • Coombs (-) Hemolytic anemia→fatigue, jaundice, ↑bilirrubin, ↑LDH, ↓Haptoglobin, hemoglobinuria
  • Thrombosis at atypical sites (ex, hepatic, portal, cerebral veins)
  • Pancytopenia
  • Breakdown of iron containing RBC→iron deposition in kidney►Hemosiderosis
313
Q

Pathogensis of paroxysmal nocturnal hemoglobinuria.

A

Mutation of phosphatidylinositol glycan class A (PIGA) gene→helps synthesize glycosylphosphatidylinositol (GPI) anchor protein→attach several cell surface proteins►CD55 (decay accelerating factor), CD59 (MAC inhibitory protein)→inactivate complement

*Absence of CD55, CD59→complement-mediated hemolysis►CD55/CD59 (-) on flow cytometry

314
Q

Treatment of paroxysmal nocturnal hemoglobinuria.

A

Eculizumab→terminal complement inhibitor

315
Q

Major antigen involved in the pathogenesis of idiopathic membranous nephropathy

A

IgG4 antibodies to the phospholipase A2 receptor (PLA2R) [transmembrane protein abundant on podocytes]

316
Q

How can you identify gallbladder hipomotility and what would be the pathologic?

A
  • Slow/incomplete gallbladder emptying in response to cholecystokinin stimulation
  • Billiary stasis→biliary sludge
317
Q

What is a pure red cell aplasia (PRCA) and its pathogenesis? Common diseases associated.

A
  • Severe hypoplasia of marrow erythroid precursor, with normal granulopoiesis and thrombopoiesis
  • Inhibition of RBC precursors by IgG autoantibodies or cytotoxic T lymphocytes. Or Parvovirus B19 infection (anti-B19 IgM in serum)
  • Associated with thymoma and lymphocytic leukemia
318
Q

Histopathological finding of esophageal squamous cell carcinoma.

A
  • Solid nests of neoplastic squamous cells with abundant eosinophilic cytoplasm and distinct borders
  • Keratin pearls
  • Intercellular bridges
319
Q

What is the disease and pathologic finding if patient has fixed, hard (rock-like), painless goiter, extended to local structures (eg, trachea, esophagus - dyspnea, dysphagia, hoarseness, chiking, coughing)? Pathogenesis.

A
  • Riedel thyroiditis→Thyroid replaced by fbrous tissue with inflammatory infltrate. Fibrosis may extend.
  • Manifestation of IgG4-related systemic disease (eg, autoimmune pancreatitis, retroperitoneal fbrosis, noninfectious aortitis).

*Lead in hypothyroidism - 3/4 of patients

320
Q

What is the antibody present in the DM type I patients and genetic association?

A
  • Glutamic acid decarboxylase (anti-GAD) antibodies→Autoimmune destruction of β cells
  • HLA-DR3 and -DR4
321
Q

Suggestive laboratory finding of alcoholic hepatitis

A
  • AST:ALT ratio >2:1 (elevation <300 U/L)
  • ↑Bilirrubin, ↑Gamma-glutamyl transferase (GGT)

*Viral hepatitis both transaminases are ↑ (usually >800 U/L), ratio <2:1

322
Q

How do you differentiate achalasia from pseudoachalasia?

A

Pseudoachalasia→more rapid progression, asymmetry in esophageal wall thickening, inability to pass an endoscope through the lower esophageal sphincter (LES), enlarge lymph nodes

*Both→↑LES pressure, absence of peristalsis

323
Q

What is principal cause of a drop of progesterone during pregnancy, and its consequence?

A

hCG deficiency→one of the most common cause of first trimester spontaneus abortion

324
Q

Disease and clinical presentation in a patient with a brain CT scan showing hypoplasia/absence of cerebellar vermis and cystic dilation of the forth ventricle with posterior fossa enlargement.

A
  • Dandy-Walker malformation
  • Infancy developmental delay, progressive skull enlargement. Cerebellar dysfunction→unsteadiness and impair muscle coordination. Non-communicating hydrocephalus (atresia of the foramina of Luschka and Magendie)
325
Q

Classical presentation and main age group affected by chiari type I and II malformation.

A
  • Chiari type I→less severe, adulthood, paroxysmal occipital headache (meningeal irritation), cerebellar dysfunction (ataxia, dizzines - compression of cerebellar tonsils)
  • Chiari type II→more severe, neonates, lumbar myelomeningocele (lower limb paralysis), non-communicating hydrocephalus (aqueductal stenosis), medulla compression (dysphagia, stridor, apnea)
326
Q

Most common gene mutation in melanoma, mechanism to cause metastatic disease and place where can make metastasis.

A
  • BRAF mutation (oncogene) V600E [valine→glutamic acid]→serine/threonine kinase→↑↑activation of signaling pathway of melanocyte growth, survival and metastasis
  • Brain metastasis→several round lesions
327
Q

Most probably diagnosis if findings are anemia, thromocytopenia, absent hematopoietic cells in bone marrow (triad). How do you expect the erythropoietin levels?

A
  • Aplastic anemia

- Anemia induced hypoxia→↑Erythoropoietin (with normal renal function)

328
Q

What disease is suggested if find decrease ApoE in circulating lipoproteins? Main clinical presentation and mechanism of disease.

A
  • Familial dysbetalipoproteinemia (type III)→↓ApoE3, ApoE4
  • Premature atherosclerosis (coronary and peripheral vascular disease), tuboeruptive and palmar xanthomas
  • ↑Chylomicrons and VLDL (impair uptake or remove by liver cells)
329
Q

Classic triad of clear cell carcinoma of kidney. Most common sites of metastasis.

A
  • Hematuria, flank pain, palpable abdominal mass→less than 10% of cases
  • 1st→Lung (in 50% of disseminated disease), 2nd→Bone

*More common non-specific symptoms→fever, malaise, anorexia, weight loss

330
Q

Paraneoplasic syndromes caused by clear cell carcinoma of kidney

A
  • ↑Erythropoietin→Erythrocytosis and polycitemia (↑Hematocrit)
  • Parathyroid hormone-related peptide→Hypercalcemia
331
Q

What is Rett syndrome? Main and classic clinical features.

A
  • Neurodevelopmental disorder mainly in girls
  • Normal development until 5-18 mo→loss motor and language skills, stereotypic hand movements
  • Deceleration of head growth→classic, early sign
  • Others: seizures, intellectual disability, austistic features, breathing abnormalities
332
Q

Cause of Rett syndrome

A

De Novo mutations in the X-linked MECP2 gene

333
Q

How can you differentiate by physical examination at birth the non-cyanotic congenital heart defects?

A
  • Ventricular septal defect→blowing, holosystolic murmur mid to lower left sternal border
  • Atrial septal defect→wide, fixed splitting of S2
  • Patent ductus arteriosus→continuous murmur
334
Q

What is the similitude between measles (rubeola) and german measles (rubella) and what finding may help you to differentiate? Type of virus of each one.

A

Both febrile maculopapular rash initiating at face and spreading to extremities and trunk

  • Measles (rubeola)→rash coalesces and darken. No postauricular lymphadenopathy→Paramyxovirus
  • German measles (rubella)→rash spread faster, doesn’t coalesce or darken. Tender postauricular and occipital lymphadenopathy→Togavirus

*Suspect them in non-vaccinated immigrant

335
Q

How do you identify histologically medullary thyroid cancer? Laboratory finding associated.

A
  • Spindle-shaped cells in an amorphous background→parafollicular calcitonin-secreting C cells
  • ↑Calcitonin

*Commonly from “cold thyroid nodule”

336
Q

Most common genetic alteration in medullary thyroid cancer

A
  • 80% sporadic→RET proto-oncogene mutation
  • 20% familial (MEN type 2 syndrome)→95% have germline mutation in RET proto-oncogene►membrane-bound tyrosine kinase receptor→cell cycle regulation
337
Q

Which finding help you differentiate small cell lung carcinoma (undifferentiated or oat cell carcinoma)?

A
  • Centrally located, strongly association with smoking
  • Round oval cells with scant cytoplasm and large hyperchromatic nuclei, in sheets or clusters. Abundant mitoses
  • Positive neuroendocrine markers→neuron-specific enolase, chromogranin, synaptophysin, Neural cell adhesion molecule (CD56), Neurofilaments
338
Q

Most common benign liver tumor and its histopahologic finding.

A
  • Cavernous hemangioma
  • Cavernous, blood-filled vascular spaces, variable size lined by single epithelial layer. Collagenous scars or fibrous nodules, thrombosis

*Bx of suspected hemangioma is nor advisable→risk of fatal hemorrhage, low diagnostic yield

339
Q

What is the screening test and confirmatory test for Celiac disease?

A
  • Screening→↑IgA against tissue transglutaminase (intestinal enzyme required for gluten metabolism)
  • DUODENAL biopsy→villus flattening, intreaepithelial lymphocyte infiltration
340
Q

In the context of cystic fibrosis, a finding of squamous metaplasia in pancreatic ducts is most likely due to a deficiency in which vitamin? why?

A
  • Pancreatic inssuficiency→fat-solubles vitamins deficiency
  • Vitamin A→maintain orderly differentiation of specialized epithelia (ex, mucus secreting columnar epithelium of ocular conjunctiva, respiratory, urinary tracts, pancreatic and other exocrine ducts)
341
Q

Possible clinical manifestations of vitamin E deficiency and pathogenesis. Most common causes associated to the deficiency.

A
  • Free radical damage of cell membranes→Hemolysis, neurologic dysfunction
  • Mimic Friedreich ataxia (same areas affected)►ataxia (spinocerebellar tracts), loss position and vibration sense (dorsal column degeneration), loss deep tendon reflexes (peripheral nerve degeneration)
  • Fat malabsorption and abetalipoproteinemia

*Also can cause infertility and ↓serum phospholipids

342
Q

What is required by an intestinal carcinoid tumor to cause carcinoid syndrome and why?

A
  • Metastasize to the liver→vasoactive substances secreted (5TH, PG, bradykinin) bypass the first-pass metabolism on it►enter to the systemic circulation
  • Local vasoactive substances secreted by intestinal tumors are metabolized by liver→prevent carcinoid syndrome

*Extrainstestinal carcinoid tumors (bronchial carcinoid)→carcinoid sx without metastasis►substances secreted directly to systemic circulation

343
Q

Key findings of Von Hippel-Lindau disease.

A

Cerebellar hemangioblastoma + congenital cysts of kidneys, liver and/or pancreas. Clear cell renal carcinoma, pheochromocytoma.

*Autosomal dominant→Deletion or mutation in VHL tumor supressor gene at chromosome 3p

344
Q

Possibles clinical presentation of Tuberous sclerosis and major complication.

A
  • Kidney, liver and pancreas cysts
  • CNS→cortical and subependymal hamartomas
  • Cutaneous angiofibromas (adenoma sebaceum), visceral cysts, other hamartomas, ash-leaf patches
  • Renal angiomyolipomas, cardiac rhabdomyomas
  • Major complication→seizures, cognitive disability

*Autosomal dominant

345
Q

Aproximately how much time pass for the interferon secretion by activated T lymphocyte during tuberculosis infection? Objectives of the IFN.

A
  • 2-4 weeks after initial infection→organism ingested by macrophages→presented to TH lymphocytes
  • Macrophage activation→epithelioid cells, improve ability to kill Mycobacterium
346
Q

What happen during the first week of Mycobacterium tuberculosis infection?

A

Alveolar macrophages phagocytes mycobacteria→Intracellular bacterial proliferation

347
Q

Diagnostic test for hereditary espherocytosis.

A

Acidified glycerol lysis test→lysis of RBC (release Hb) when incubated in hypotonic saline with glycerol►positive

*Spherocytes have increased osmotic fragility→decreased surface area to volume ratio

348
Q

Complications of hereditary spherocytosis.

A
  • Pigmented gallstones

- Aplastic crisis by parvovirus B19 infection

349
Q

Which coronary artery occlusion can cause infranodal AV block mobitz type II or 3rd degree?

A

Left anterior descending (LAD)→blood supply interventricular septum and anterior wall of left ventricle

*Most nodal AV blocks are due occlusion of Right coornary artery branches→supply SA and AV nodes

350
Q

Which is the structure that cause intraventricular hemorrhage? Function of that structure.

*Longterm consequence

A
  • Germinal matrix→contain numerous thin walled vessels lacking glial fibers
  • Highly cellular and vascularized layer in the subventricular zones from which neurons and glial cells migrate out during brain development

*Common complication of prematurity→longterm of neurodevelopmental impairment

351
Q

Which findings may suggest you the second most common cause of acute pancreatitis if gallstones are ruled out?

A

Alcohol abuse

  • > 2 AST:ALT ratio
  • Macrocytosis→due vitamin (folate deficiency) or independently of it, liver disease, direct toxicity of OH on bone marrow
352
Q

What mechanism of disease cause a fluctuating muscle weakness that worsens over the day, mainly affecting extraocular muscles (ptosis, diplopia), bulbar muscles (fatigable chewing, dysphagia) in a patient with thymic abnormalitie (thymic hyperplasia, thymoma)?

A

Myasthenia gravis→antibodies against nicotinic Ach receptor

*Anterior mediastinal mass of chest x-ray→thymoma or thymic hyperplasia

353
Q

Overexpression of which gene is associated with aggressive and good progonosis in breast cancer?

A
  • ↑HER2→transembrane glyprotein, tyrosin kinase, ↑cell proliferation►positivity►→poorly differentiated, rapidly growing tumor; response to anti-HER2 monoclonar antibodies (trastuzumab)
  • ↑BCL-2→good prognosis in breast cancer (lympoma is bad prognosis)
  • ↑Estrogen and progesterone receptors→good prognosis→sensitivity to tamoxifen or aromatase inhibitors
354
Q

What is cutaneous small vessel vasculitis and causes associated? Histopathological finding.

A
  • Leukocytoclastic vasculitis→only affects the skin►drugs (penicilins, cephalosporins, sulfonamides, phenytoin, allopurinol), pathogens (ex, hepatitis B or C)
  • Perivascular inflammation of small blood vessels, fibrinoid necrosis, neutrophils, fragmented neutrophilic nuclei

*Nonblanching palpable purpura in lower extremities

355
Q

Clinical presentation of leukocytoclastic vasculitis.

A

Nonblanching palpable purpura, usually in lower extremities

356
Q

Higher risk of malignant potential of adenomatous polyps?

A
  • Degree of dysplasia
  • Villous adenoma>tubular adenoma
  • Size >4cm
357
Q

Which coagulation test may be altered in antiphospholipid syndrome?

A
  • Activated partial thromboplastin time (aPTT)→paraxodically ↑ because patients have hypercoagulable state
  • Antiphospholipid antibodies activate phospholipid-dependent coagulation pathways
358
Q

How is minimized the work of breathing in diseases with increase elastic resistance (pulmonary fibrosis, pulmonary edema, ARDS), and increase airflow resistance (COPD, asthma)?

A
  • ↑Elastic resistance→↑respiratory rate and ↓tidal volume (fast, shallow breaths)
  • ↑Airflow resistance→↓repiratory frequency (deep) and ↑tidal volume (deep, slow breaths)
359
Q

What should you assess and suspect in a pregnant woman with unexplained weight loss, iron deficiency anemia and craving ice?

A

Pica→compulsie consumption of a nonfood and/or non-staple food for >1 month (also in school children)→earth/soil rich substances, raw starch (flour or conrstarch), ice

*Association with other nutritional deficiencies: iron, zinc, anemia of any etiology. At any stage of pregnancy.

360
Q

Which fusion protein that promotes carcinogenesis is the molecular pathophisiology of non-small cell lung cancer?

A

Echinoderm microtubule-associated protein like 4 + anaplastic lymphoma kinase (EML4-ALK)→inversion of short arm of chromosome 2►constitutively active tyrosin kinase

*Young non-smokers, with adenocarcinoma without mutations at EGFR and K-ras genes

361
Q

Treatment for the lung cancer associated with fusion protein EML4-ALK. What is the neplasia?

A
  • Crizotinib→protein kinase inhibitor

- Non-small cell lung cancer

362
Q

Treatment for the neoplasia associated with fusion protein BCR-ABL. What is the neoplasia?

A
  • Imatinib→protein kinase inhibitor

- Chronic myelogenous leukemia

363
Q

Which disease can cause a combined upper motoneuron and lower motoneuron lesions?

A

Characteristic of amyotrophic lateral sclerosis (Lou Gehring disease)

364
Q

How do you differentiate myasthenia gravis from lambert-eaton syndrome (LEMS)?

A
  • LEMS→hyporeflexia, areflexia, autonomic symptoms (dry mouth, impotence), classic incremental response to repetitive stimulation
  • LEMS associated with paraneoplasic syndrome (small cell lung cancer)
  • MG→thymoma, thymic hyperplasia
365
Q

Marker for small cell carcinoma of the lung

A

Neuroendocrine differentiation→Neuroendocrine markers:

  • Neural cell adhesion molecule (NCAM or CD56)
  • Neuron-specific enolase
  • Chromogranin
  • Synaptophysin
  • Also neurosecretory granules
  • Most aggressive type of lung cancer
366
Q

Which structures obstruct the left ventricular outflow in hypertrophic cardiomyopathy?

A

Mitral valve leaflet and interventricular septum→abnormal systolic anterior motion of the mitral valve toward a hypertrophied IV septem

367
Q

Why does alcoholic hepatic steatosis occur?

A

Active 2 major alcohol metabolic ezymes→alcohol dehydrogenase and aldehyde dehydrogenase→↑NADH→↓free fatty acid oxidation (also gluconeogenesis)

368
Q

Triad of hemolytic uremic syndrome

A
  • Microangiopathic hemolytic anemia
  • Thrombocytopenia
  • Acute kidney injury
369
Q

Moldy grains in china that cause a specific G to T mutation at p53 gene increase the risk of which disease?

A

G:C→T:A transversion in codon 249 of p53 gene→ Liver cancer→hepatocellular carcinoma

*Aspergillus flavus and parasiticus→grow in corn, soybeans, peanuts→↑aflatoxins►classified as A1, B2, G1, G2, B1 (most common and most toxic)

370
Q

Histologic findings of membranous glomerulopathy.

A
  • Diffuse increased thickness of the glomerular basement membrane, no cellularity (LM)
  • “Spike and domme” appearance on methenamine silver stain
  • Granular deposits of IgG and C3 (IF)
371
Q

What could be the consequence to the exposition to radioactive iodine-131 and can you prevent it?

A
  • 131-I taken by thyroid gland→radiation induced hypothyroidism or thyroid carcinoma
  • Potassium iodide→competitively inhibit radioactive 131-I from entering thyroid follicular cells, (-) iodine organification (Wolff-Chaikoff effect), ↓thyroid hormone release
  • Nuclear reactor accidents, breach of containment at waste disposal sites, atomic bomb
372
Q

What is the Menetrier disease?

A

Overproduction of TGF-alpha→potent stimulator of epithelial cell growth►mucosal cell hyperplasia, gastric fold enlargement, but hypoplasia of parietal/chief cells (glandular atrophy, ↓gastric acid)

373
Q

Which is the vessel in the body that has the lowest oxygen content and why?

A

Coronary sinus→myocardial oxygen extraction is very high (resting myocardium extracts 60%-70% O2 from blood)►cardiac venous blood is the most deoxygenated

374
Q

What type of gastrointestinal peptic ulcers are associated with lower and higher risk of cancer?

A
  • Duodenal ulcers→peptic ulcer Helicobacter pylori or NSAIDs, (>95% of ulcers), very rare malignant, don’t biopsy required
  • Gastric ulcers→can be malignant (MALT lymphoma, gastric adenocarcinoma, ↑risk with H pylori infection
375
Q

Two most common mutations responsible of hypertrophic cardiomyopathy.

A

Autosomal dominant disorder caused by mutations in sarcomere genes

  • Cardiac myosin binding protein C
  • Cardiac beta myosin heavy chain
376
Q

Histologic findings at lung of chronic and acute left heart failure.

A
  • Acute heart failure→transudate in alveolar lumen and lung interstitium
  • Chronic heart failure→hemosiderin-containing macrophages (siderophages, “heart failure cells”)
377
Q

Most probably etiology in a immunocompromised patient, fever, nodules or cavitary lesion at lungs (no parenchhymal infiltrate), ring-enhancing lesion (brain abscess), sputum show gram (+) branching rods. Another clinical presentation and features of the organism.

A
  • Nocardia
  • Cutaneuos involvement
  • Aerobic, catalase (+), partially acid fast
378
Q

Treatment of choice in nocardiosis

A

TMP-SMX

379
Q

Mechanism or pathogenesus that produce false diverticula (ex, diverticulosis).

A

Pulsion→strained bowel movements (ex, chronic constipation)→↑intraluminar pressure►herniate mucosa and submucosa through areas of focal weakness in the muscularis

380
Q

Clinical presentation and most common location of diverticulosis

A
  • Sigmoid colon, >60 years, most asymptomatic or hematochezia
  • Inflammation→Diverticulitis►left lower quadrant abdminal pain, low grade fever, constipation or diarrhea
381
Q

Pathogenesis of achondroplasia. Which bones are affected?

A

Point mutation in FGFR3 gene→exagerated inhibition of chondrocyte proliferation in growth plates→↓endochondral ossification in long bones
- Flat bones are not affected→intramembranous ossification

*Fibroblast growth factor receptor 3→normal, restrained inhibition of endochondral ossification

382
Q

Inheritance of achondroplasia

A
  • 90% mutations are new

- 10% mutations inherited→autosomal dominant

383
Q

What bones affect the growth hormone deficiency? causes of this alteration.

A
  • Hypothalamic or pituitary insults (tumor, ischemia)→↓GHRH and GH►proportional short stature (long and flat bones affected)
  • Achondroplasia just long bones are affected→disproportionated short stature
384
Q

Skin alteration associated with gluten enteropathy.

A

Dermatitis herpetiformis→4th-5th decade of life, clusters of puritic vesicles and plaques on the buttocks and extensor surfaces of extremities

385
Q

Diseases associated with atopic dermatitis (eczema) and how are the lesions?

A
  • Pruritus and erythematous, weeping/crusted papules in response to certain environmental antigens in children, often by 5 years.
  • Older children→lichenification in flexural distribution
  • Allergic triad→AD, asthma, allergic rhinitis
386
Q

What does migratory superficial thrombophlebitis suggest and why? How does it call?

A

Visceral adenocarcinomas→pancreas, lung, colon►paraneoplastic production of thromboplastin-like susbtance→chronic intravascular coagulations, disseminate and migrate
*Trousseau syndrome

387
Q

Pituitary Hormone levels in primary ovarian failure

A

↑LH, ↑FSH

*Turner syndrome one of the most common cause

388
Q

What is the vaginal adenosis and its cause? What is the consequece?

A
  • Replacement of the squamous epithelium with glandular columnar epithelium
  • Female children of woman exposed to diethylstilbestrol (DES) during pregnancy
  • Precursor of clear cell adenocarcinoma of the vagina
389
Q

Classical histology finding of Whipple disease. Causal agent of it.

A
  • Small intestine mucosa containing enlarged, foamy macrophages in lamina propia packed with rod-shaped bacilli and PAS (+), diastase-resistant granules (lysosomes and partially digested bacteria)
  • Tropheryma whipplei
390
Q

Clinical presentation of whipple disease

A

Middle age caucasian males

  • Smal intestine→malabsorption with diarrhea and weight loss
  • Joints→arthropathy, polyartritis
  • CNS, pychiatric
  • Cardiac
  • Chronic course
391
Q

Mention the 3 most common brain tumors in children

A
  1. Pilocytic astrocytoma→low-grade neoplasm, good prognosis►cerebellum (also cerebral hemispheres)
  2. Medulloblastoma→most common malignant type►cerebellum
  3. Ependymoma→hydrocephalus, microscopic gland-like structures (“Rosettes”)
392
Q

How can you differentiate medulloblastoma and pilocytic astrocytoma? What do they have in common?

A
  • Both located mainly in cerebellum
  • Pilocytic astrocytoma→macro: cystic with a tumor nodule (active part) protruding from the wall►solid and cystic component in imaging. micro: pilocytic astrocytes, rosenthal fibers
  • Medulloblastoma→macro: solid. micro→sheets of small blue cells with hyperchromatic nuclei and scant cytoplasm
393
Q

What is the most important criteria to determine the prognosis of colorectal adenocarcinoma?

A

Tumor stage→extent of tumor expansion

  • Confined to mucosa→90% survival at 5 yrs (A)
  • Muscular layer→70-80% at 5 yrs (B)
  • Lymph node (C), distant metastasis (D)→poor prognosis

*Tumor Grade→degree of differentiation (well to anaplastic) - not importanta for prognosis in colon cancer

394
Q

In the setting of microangiopathic hemolytic anemia with schystocytes and thrombocytopenia how do you differentiate from disseminated intrascular coagulation and TTP or HUS?

A
  • DIC→↑PT, ↑PTT→cascade coagulation activation

- HUS and TTP→normal PT, PTT

395
Q

What may suggest a nonbacterial thrombotic endocarditis?

A

Marantic endocarditis→Advanced malignancy (mucinous adenocarcinomas), chronic inflammatory disorders (antiphospholipid syndrome, SLE [Libman-Sacks endocarditis]), DIC (sepsis)

396
Q

Clinical presentation of glucagonoma

A
  • Necrolytic migratory erythema→maybe due to ↓levels of amino acids
  • Diabetes Mellitus
  • GI symptoms→diarrhea, anorexia, abdominal pain
  • Normocytic normochromic anemia (chronic disease)
397
Q

Most probably radiologic finding in patient with a cholesterol-mass lodged at the ileocecal valve, and why?

A
  • Pneumobilia (air in the biliary tree)→retrograde gas through fistula into gallbladder
  • Dilated bowel loops with air-fluid levels→small bowel obstruction
  • Gallstone ileus→large gallstone (>2,5 cm) erodes into intestinal lumen by cholecystoenteric fistula►mechanical obstruction of the intestine→small bowel obstruction

*True ileus→nonmechanical disruption on intestinal motility

398
Q

Early menarche (before age of 12) is associated with high risk of which type of cancers?

A

Breast and endometrial cancers

399
Q

Most common cause of sudden cardiac death after miocardial infarction.

A

Malignant ventricular arrhythmias→sustained ventricular tachycardia, ventricular fibrilation (most common)

400
Q

Highest risk factor associated with metastasis of malignant melanoma.

A

Active vertical growth
- Breslow depth→distance from the epidermal cell layer to the deepest visible melanoma cells►most important prognostic indicator

401
Q

Actinic keratoses is associated with higher risk of which neoplasia?

A

Premalignant lesion of invasive squamous cell carcinoma (several years, small %)→↑lesion size and thickness, dermal invasion, metastatic potential

*Result from chronic sun exposure

402
Q

Laboratory findings of postreptococcal glomerulonephritis.

A
  • ↑anti-streptolysin O, anti-DNase B, anti-cationic proteinase
  • ↓C3 (activation of alternative complement pathway)
  • C4 normal or slightly ↓ (classical pathway)
  • Cryoglobulins
403
Q

Most common non-glial brain tumor of children

A

Craniopharyngioma

404
Q

Diseases associated with increased and decreased pulse pressure.

A
  • ↑Pulse pressure→arteriosclerosis, aortic regurgitation

- ↓Pulse pressure→aortic stenosis, mitral stenosis

405
Q

Genetic cause of DiGeorge syndrome and velocardiofacial syndrome. How do you diagnose it?

A
  • Chromosome 22q11.2 microdeletion
  • Fluorescent in-situ hybridization→genes of interest are hybridized with fluorescently labeled DNA probed (lack of fluorescent signal→microdeletion)
406
Q

What structure and what type of hearing loss cause Noise-induced hearing loss (chronic)? Bilateral symptoms.

A
  • High frequency hearing loss

- Trauma of stereociliated hair cells of the organ of corti

407
Q

How can you distinguish the infectous esophagitis in immunocompromised patients?

A
  • Candida: white pseudomembrane
  • HSV-1: punched-out ulcers, glassy eosinophilic intracnuclear inclusions within epithelial cells
  • CMV: linear ulcers, large basophilic intranuclear inclusions (“owl-eyes”) with granular cytoplasmic inclusions, markedly enlarged cell size
408
Q

Which neoplasm resemble microscopically Kaposi sarcoma?

A

Angiosarcoma→skin (radiation induced chronic lymphedema), liver (polyvinyl chloride exposure, thorotrast dye, arsenic [persticides])
*Proliferating stromal cells and endothelium (spindle shape) creating vascular channels with blood cells. CD31 (+).

409
Q

Similar infectious condition to Kaposi sarcoma also seen in AIDS patients.

A

Bacillary angiomatosis→blood vessel proliferation and violaceous nodules.
*Bartonella henselae→cat scratch or bite

410
Q

Through which cell does shigella species invades the gastrointestinal mucosa? Most common species causing shigellosis in industrialized countries.

A
  • Microfold (M) cells in ileal Peyer patches (endocytosis)→lyses endosome►spreads laterally into other epithelial cells→cell death, ulceration►hemorrhage, diarrhea
  • Shigella sonnei (80%)
411
Q

Common presentation of sickle cell disease in first few years of life.

A

Dactylitis→hand-foot syndrome►small infarctions in the bones of the extremities→swelling, tenderness, warmth

*Affected bones still contain hematopoietic bone marrow in first few years

412
Q

What is DRESS syndrome and its probably cause?

A
  • Drug Reaction with Eosinophilia and Systemic Symptoms (DRESS)→rare and potentially life-threatening drug reaction occurring 2-4 wks after drug exposure
  • Anticonvulsivants (phenytoin, carbamazepine), allopurinol, sulfonamides, Antibiotics (minocycline, vancomycin)
413
Q

Mechanism of DRESS syndrome

A

Drug induced herpesvirus reactivation followed by clonal expansion of T cells that cross-react with the drug

414
Q

Clinical presentation of Bell’s palsy

A

Unilateral facial paralysis, decreased tearing, hyperacusis, loss of taste over anterior 2/3 of tongue

*Impair function of VII CN

415
Q

Most common thyroid cancer presenting as a solitary cold nodule

A

Follicular carcinoma

416
Q

Localized bony back pain, low grade fever, recent bacteremia (Staphylococcus) suggest what disease? Initial studies that should be done.

A
  • Vertebral osteomyelitis
  • Blood cultures and MRI

*Also suspect if new neurologic findings and fever with or without back pain

417
Q

What type of tumor is a seminoma? Other examples of that type. Tumor marker of each of them.

A
  • Germ cell tumors (95% of all testicular neoplasm)→from the cell line maturing into sperm cells
  • Seminoma (placental alkaline phosphatase [PLAP])
  • Yolk sac (endodermal sinus) tumor (↑AFP)
  • Choriocarcinoma (↑hCG)
  • Teratoma
  • Embryonal carcinoma→Pure (rare)►↑hCG, normal AFP, ↑AFP when mixed (common)
418
Q

Mention the testicular non-germ cells tumors. Important features of them.

A
  • 5% of all testicular tumors. Mostly benign
  • Leydig cell→ Reinke crystals (eosinophilic cytoplasmic inclusions),gynecomastia in men, precocious puberty in boys
  • Sertoli cell→may produce androgen and/or estrogens
  • Testicular lymphoma→Most common testicular cancer in older men, No 1°, from metastatic lymphoma
419
Q

Medial medullary syndrome. Clinical presentation and cause of it.

A
  • Infarct of paramedian branches of Anterior Spinal Artery and/or vertebral arteries
  • Contralateral paralysis (upper and lower limbs)→Lateral corticospinal tract
  • ↓Contralateral proprioception, tactile and vibration sensation→Medial lemniscus
  • Ipsilateral hypoglossal dysfunction (tongue deviates ipsilaterally)→Caudal medulla (hypoglossal nerve)
420
Q

What is the Plummer-Vinson syndrome?

A

Iron deficiency anemia, dysphagia (esophageal web formation), koilonychia, shiny red tongue (atrophy of lingual papillae)

421
Q

Genetic association with Neuroblastoma. Urine markers and biopsy features.

A
  • Genetic association→N-myc oncogene
  • Uirne markers→↑Homovanillic acid (HVA), ↑Vanillylmandelic acid (VMA)
  • Biopsy→Homer-Wright rosettes (also in medulloblastoma), Bombesin and Neuron specific enolase) NSE ⊕
422
Q

Which acid base disturbance could cause salicylate poisoning? and why?

A
  • Primary respiratory alkalosis→overstimulation of the CNS respiratory center
  • Metabolic acidosis (↑anion gap)→mitochondrial toxin that uncouples oxidative phosphorylation►tissue hypoxia and lactic acidosis
423
Q

Microscopic findings of alcoholic hepatitis

A
  • Fatty change with neutrophil infltration
  • Mallory bodies→Damaged cytokeratin intermediate flaments
  • Perivenular fbrosis
424
Q

Dermatomyositis can be a paraneoplastic syndrome of which tumors?

A

Ovarian, lung or colorectal cancers

425
Q

When do you suspect a pineal gland mass?

A
  • Obstructive hydrocephalus→aqueductal stenosis►papilledema, vomiting, headache
  • Parinaud (dorsal midbrain) syndrome→compression pretectal region of midbrain (superior colliculus)►upward gaze palsy, downward gaze preference, bilateral eyelid retraction (Collier sign), light-near dissociation (absent pupillary light reflex), impaired convergence
426
Q

Most common pineal mass and its locations

A

Germinoma→midline malignant tumor from embryonic germ cells

  • Pineal gland→Obstructive hydrocephalus and parinaud syndrome
  • Suprasellar region (less common)→NO parinaud syndrome; endocrinopathies
427
Q

Why squat position relieve cyanosis and dyspnea in tetralogy of fallot (“Tet spell” - acute hypoxemia)?

A

Increase systemic vascular resistance (SVR) (without change pulmonary vascular resistance [PVR])→↑ratio SVR:PVR→right ventricular output goes to pulmonary circulation→oxygenation

428
Q

What ratio volume increase in COPD?

A

↑RV more than ↑TLC►↑RV/TLC

*Air trapping in the lungs

429
Q

Microscopic findings of glioblastoma multiforme

A
  • Pseudopalisading necrosis→foci of necrosis surrounded by tumor cells
  • Neovascularization

*Most common primary brain neoplasm in adults

430
Q

Vasculitis with fibrinoid necrosis and intense neutrophil infiltration after exposure to antigenic heterologous proteins or nonhuman immunoglobulins (venom toxins). Laboratory finding. Clinical presentation.

A
  • Serum-sickness→tissue deposition of circulating immune complexes (type III hypersensitivity)
  • Hypocomplementemia (↓C3, C4)
  • Fever, pruritic skin rash, arthralgia 7-14 days after exposure. Lymphadenopathy and proteinuria.
431
Q

Which drug can improve or control Grave’s ophtalmopathy and its mechanism?

A

High-dose glucocorticoids→antiinflamatory effect→↓inflammatory cells infiltration

*Antithyroid drugs don’t have direct effect on ophtalmopathy

432
Q

Lead poisoning symptoms and signs. Risk factor.

A
  • Neurologic→cognitive impairment, irritability
  • GI→constipation, abdominal pain
  • Renal→interstitial nephritis
  • Hematologic→sideroblastic anemia
  • Lead lines in gingivae (Burton llines) and metaphyses of bones
  • ↑risk in old houses with chipped paint
433
Q

Diagnosis of lead poisoning

A
  • ↑blood lead level

- ↑urine d-aminolevulininc acid (heme synthesis substrate)

434
Q

Which physiologic factor determines the ventricular contraction rate during atrial fibrillation?

A

Transmission of abnormal atrial impulses through the AV node

435
Q

Main risk factor to disseminated and superficial candidiasis

A
  • Low T lymphocytes→superficial candidadiasis (ex, HIV)
  • Low Neutrophils→disseminated candidiasis (candidemia, endocarditis; in neutropenic, cancer with chemotherapy, impairment of phagocytosis)
436
Q

How can you suspect a cholesteatoma?

A
  • Pearly mass behind behind the tympanic membrane
  • Painless otorrhea
  • Lytic enzymes→erode through auditory ossicles→conductive hearing loss
  • ↑Size mass to vestibular apparatus or facial nerve→vertigo or facial palsies
437
Q

What is the composition of cholesteatoma? How is it produced?

A

Accumulation of squamous cell debris

  • Congenitally
  • Primary→chronic negative pressure in middle ear→retraction pockets in tympanic membrane→cysti►accumulation
  • Secondary→Infection, trauma, surgery►epithelium migrates to middle ear
438
Q

Specific features of hyperthyroidism due to Graves disease

A

Pretibial myxedema and exophtalmos

*Autoimmune response anti-TSH receptor→accumulation of GAG’s within affected tissues

439
Q

Cardiovascular manifestation of syphilis, pathogeneis and phase of the disease when it happens.

A
  • Tertiary syphilis→vasa vasorum endarteritis and obliteration►inflammation, ischemia, weakening of the adventitia→aneurysmal dilation of thoracic aorta
  • Aortic regurgitation (dilation of aortic annulus), mediastinal widening, (+) FTA-ABS
440
Q

Ring enhancing lesions in brain of a HIV patients, most commonly correspond to which diseases?

A
  1. Toxoplasmosis

2. Primary central nervous system lymphoma (PCNSL)

441
Q

First neurological deficit caused by syringomyelia

A

Damage white ventral comissure→decussation of spinothalamic tract►bilateral loss of pain and temperature sensation in upper extremities (arms and hands - “cape-like”)

*Fine touch sensation is preserved.

442
Q

Later involvement and neurological deficit in syringomyelia (extension of syrinx)

A

Ventral horns damage→flaccid paralysis and atrophy of intrinsic muscles of hand

443
Q

What is the sheehan syndrome? Most common presentation.

A
  • Ischemic necrosis of pituitary gland→↑Estrogen pregnancy→hyperplasia f pituitary gland (lactotrophs) without proportional ↑ in vascular supply
  • Postpartum hemorrhage→ischemic injury
  • Panhypopituitarism→fail to lactation (↓prolactin), central hypothiroidism, adrenal insufficiency (↓cortisol)
444
Q

Cause and location of lacunar infarcts.

A
  • Hypertensive arteriolar sclerosis→chronic hypertension►lipohyalinosis, microatheroma, hardening/thickening of vessel wall►narrowing of lumen, ↑thrombosis
  • Deep brain structures→basal ganglia pons
  • Subcortical white matter→internal capsule, corona radiata
445
Q

Histologic features of myxoma and clinical features.

A
  • Scattered cells within mucopolysaccharide stroma, abnormal blood vessels, hemorrhaging (↑VEGF)
  • 80% in left atrium, constitutional symptoms (↑IL-6)
  • Mid-diastolic rumbling murmur at apex, positional dyspnea, large pedunculated mass, embolic symptoms
446
Q

Mechanism of toxicity of carbon tetrachloride (CCL4). Histology finding.

A

Free radical injury
- Oxidation by P450 oxidase system in liver→free radical CCL3►react with structural lipids of cell membranes►H2O2►lipid peroxidation (vicious cycle)→hepatocyte necrosis

447
Q

Which vitamin is useful during episode of measles and why?

A
  • Measles→↓↓Vitamin A stores→↑risk of keratitis and corneal ulceration
  • Prevent and Tx ocular complications, ↓risk of other complications (pneumonia, encephalitis), ↓recovery time, length of hospital stay
448
Q

Pathogenesis of pulmonary hypertension in left ventricular heart failure.

A
  • Pulmonary venous congestion→↑pulmonary capillary and arterial pressure►endothelial damage, capillary leakage of proteins into interstitium→↓NO (vasodilation), ↑Endothelin (vasoconstrictor)→↑vascular tone [Vasoconstriction]
  • Remodeling of pulmonary vasculature→↑smooth muscle proliferation (medial hypertrophy), collagen, elastase deposition (intimal thickening and fibrosis)
449
Q

How does paraneoplastic cerebellar degeneration occur?

A
  • Immune response against tumor cells crossreacts with Purkinje neurons antigens→acute onset rapid degeneration of cerebellum
  • Antibodies in serum→Anti-Yo, anti-P/Q, anti-Hu
450
Q

Most common clinical presentation and histology findings of paraneoplastic cerebellar degeneration.

A
  • Small cell lung cancer, breast, ovarian, uterine
  • Progressively worsening dizziness, limb and truncal ataxia, dysarthria, visual disturbances (diplopia, oscillopsia)
  • Extensive purkinje cell degeneration
451
Q

Most likely vessels affected and spared in polyarteritis nodosa

A
  • Spares pulmonary arteries and rarelly involves bronchial arteries
  • Affect (ischemia, infarction or hemorrhage) kidneys, heart, liver, GI tract→propensity for bead-like aneurysm (mesenteric circulation)

*1/3 patients have cutaneous manifestations→livedo reticularis, palpable purpura

452
Q

Mechanism of tissue damage and resultant pulmonary abcess formation secondary to lobar pneumonia

A

Lysosomal enzymes release from neutrophils and macrophages

*Local suppurative collections within lung parenchyma→necrosis of surrounding tissue

453
Q

Content and lining of pancreatic pseudocyst.

A
  • Fluid rich in enzymes and inflammatory debris
  • Walls of granulation tissue and fibrosis→most common located in lesser peritoneal sac

*True cyst→lined by epithelial cells

454
Q

Etiology and pathogenesis of Abetalipoproteinemia

A
  • Loss function mutation of Microsomal triglyceride transfer protein (MTP) gene→chaperone for proper folding of apo B, transfer lipids to Chylomicrones and VLDL
  • Impaired formation of apolipoprotein B (apo-B) containing lipoproteins→Chylomicrons (dietary lipids - Apo B-48), VLDL (endogenous lipids - Apo B-100)
455
Q

Clinical presentation and histologic findings of abetalipoproteinemia

A
  • Clear or foamy cytoplasm of enterocyte at tip of villi→accumulation of lipids
  • First year of life, malabsorption (abdominal distention, foul smelling stool), ↓↓TG, cholesterol, absent apo B, deficiency of fat-solluble vitamins (Vit E), acanthocytes, neurologic abnormalities (progressive ataxia, retinitis pigmentosa)
456
Q

Renal complication of infective endocarditis, how does it occur?

A

Deposition of immune complexes in the glumerular capillary wall→diffuse, proliferative glomerulonephritis►Nephritic syndrome→acute renal insufficiency

457
Q

Most probable diagnostic in an infant with solid, hypervascular mass at the upper pole of a kidney. Pathogenesis and clinical presentation.

A
  • Nephroblastoma (Wilms tumor)→Contains embryonic glomerular structures
  • Large, palpable, unilateral flank mass and/or hematuria
  • “Loss of function” mutations of tumor suppressor genes WT1 or WT2 on chromosome 11
458
Q

Histology finding of carbon tetrachloride (CCL4) toxic liver injury.

A

As CLL3 is formed by P450, it blocks come out fat of liver►irreversible damage that swell RER→detach ribosomes→↓protein synthesis→↓lipoproteins►Fatty change of liver

459
Q

What is the consequence of incomplete recanalization of proximal ureter and pathologic changes in ultrasonography?

A

Congenital ureteric stenosis→collecting system and pelvis of the affected kidney dilated and the renal cortex compressed. The left and right ureters are not dilated.